You are on page 1of 65

Telegram PT Group Discussion 2019 (Geography)

Q: The latitudinal and longitudinal extent of India are roughly about 30 degrees but the distance
measured from North to South is much more than distance measured from East to West. Why?
(a) Distance between two longitudes increases towards the poles whereas the distance between two
latitudes decreases towards the pole.
(b) Distance between two longitudes increases towards the poles whereas the distance between two
latitudes remains almost the same everywhere.
(c) Distance between two longitudes decreases towards the poles whereas the distance between two
latitudes remains almost the same everywhere.
(d) While latitudes vary from 0 to 90 degree North and South, longitudes vary from 0 to 180 degree East
and West.

Answer: C
The latitudinal and longitudinal extent of India, are roughly about 30 degrees, whereas the actual
distance measured from north to south extremity is 3,214 km, and that from east to west is only 2,933
km. This difference is based on the fact that the distance between two longitudes decreases towards the
poles whereas the distance between two latitudes remains almost the same everywhere.

Q: Consider the following cities:


1. Udaipur
2. Bhubneshwar
3. Nagpur
4. Jhansi
In which of the above cities are you likely to observe an overhead sun, twice a year?
(a) 1, 2 and 3 only
(b) 1 and 4 only
(c) 2 and 3 only
(d) 2, 3 and 4 only

Answer: C
o Cities located south of Tropic of Cancer will observe an overhead sun, twice a year.
o Udaipur and Jhansi are located north of Tropic of Cancer. Also, Jhansi can be eliminated because
Tropic of Cancer does not pass through Uttar Pradesh.
o Nagpur and Bhubneshwar are located south of Tropic of Cancer. Therefore, (c) is the correct answer.

Q: You are walking along the McMohan Line from west to east
1. If you jump back to the Indian side, you will find yourself in Sikkim.
2. The latitude passing from your location will cut through Nagaland and Rajasthan.
3. At the end of your journey along the line you may end up in Myanmar.
Which of the of the statements given above is/are correct?
(a) 1 only
(b) 1 and 2 only
(c) 2 and 3 only

© APTI PLUS ACADEMY FOR CIVIL SERVICES


(d) 3 only

Answer: d
The McMohan Line runs from the -junction
tri of India, Bhutan and China to the tri-junction of India,
China and Myanmar. It's eastern sector of the international border between India and China along the
state of Arunachal Pradesh.
o Statement 1 is incorrect: If you jump back to the Indian side, you will find yourself in Arunachal
Pradesh.
o Statement 2 is incorrect: The latitude passing from your location will cut through Rajasthan but not
Nagaland.
o Statement 3 is correct: At the end of the line there is tri-junction of India, China and Myanmar

Q: Tropic of Cancer passes through which of the following states in India?


1. Rajasthan
2. Bihar
3. Jharkhand
4. Tripura
Select the correct answer using the code given below.
(a) 1, 2 and 3 only
(b) 1, 3 and 4 only
(c) 2 and 4 only
(d) 1, 2, 3 and 4

Answer: B
Tropic of Cancer does not pass through Bihar. It passes through Mizoram, Tripura, West Bengal,
Jharkhand, Chattisgarh, Madhya Pradesh, Rajasthan and Gujarat.

Q: Indian Standard Meridian passes through which of the following states in India?
1. Uttar Pradesh
2. Madhya Pradesh
3. Orissa
4. Tamil Nadu
Select the correct answer using the code given below.
(a) 1, 2 and 3 only
(b) 1, 3 and 4 only
(c) 2 and 4 only
(d) 1, 2, 3 and 4

Answer: A

 1, 2 and 3 are correct. India standard Meridian (82o30 E meridian) passes through UP,
MP, Chhattisgarh, Orissa and Andhra Pradesh.

© APTI PLUS ACADEMY FOR CIVIL SERVICES


Q: Which one of the following pairs of islands is separated from each other by the 'Eight Degree
Channel?
a. Andaman and Nicobar
b. Nicobar and Sumatra
c. Maldives and Lakshadweep
d. Sumatra and Java

Answer: c
Lakshadweep group separated from Maldives by Eight Degree Channel

Q 7: Arrange the following countries in the decresing order of length of India's border shared with
them:
1. Bangladesh
2. China
3. Myanmar
4. Afghanistan
Select the correct answer using the code given below.
a. 1-2-3-4
b. 1-3-2-4
c. 2-1-4-3
d. 2-1-3-4

Answer: a

Q:Indira Point is located in:


(a) Andaman group islands
(b) Nicobar group islands
(c) Laccadive group islands
(d) Minicoy group islands

© APTI PLUS ACADEMY FOR CIVIL SERVICES


Answer: b
Indira Point (Pygmallion point) is the southern-most point of India territory(6.5° N), which is located in
Great Nicobar. It is the southernmost point of India's territory

Q:Consider the following statements.


1. India is separated from Sri Lanka by Gulf of Mannar.
2. India and Bhutan are connected through Bomdila Pass.
3. Nepal shares the longest land frontier with India.
Select the correct statement(s) using the codes given below
a) 1 and 2 only
b) 2 and 3 only
c) 1 and 3 only
d) 1, 2 and 3
Answer: c

Statement 1 is correct. The Gulf of Mannar is a large shallow bay forming part of the Laccadive Sea in
the Indian Ocean. It lies between the southeastern tip of India and the west coast of Sri Lanka, in the
Coromandel Coast Region.
Statement 2 is incorrect. Bomdila Pass connects Arunachal Pradesh with Lhasa (the capital of Tibet). This
‘mountain pass’ is situated to the east of Bhutan in the Greater Himalayas in Arunachal Pradesh at an
elevation of about 2217 mabove sea level. Owing to the snowfall and adverse weather in the winter
season it remains closed.
Statement 3 is correct. Nepal shares its boundary with two countries i.e. India and China. Out of
that Nepal has largest border with India, as compared to China. Hence, the statement 3 is correct

Q: Consider the following statements:


1. Kashmir valley lies between Great Himalayas and Ladakh range.
2. Jhelum river flows through Kashmir valley.
3. Tsomoriri and Pangong Tso are salt water lakes found in Ladakh.
Which of the statements given above is/are correct?
(a) 2 only
(b) 1 and 3 only
(c) 2 and 3 only
(d) 1, 2 and 3
Answer: C

Explanation
Statement 1 is incorrect: Kashmir Valley lies between Pir Panjal range and Great Himalayan range .
Ladakh range lies further north of Great himalayan range.
Statement 2 is correct : Jhelum, one of the main tributaries of Indus flows through Kashmir valley.
Statement 3 is correct: Both Tsomoriri and Pangong Tso are salt water lakes found in Ladakh region of
Jammu and Kashmir.

© APTI PLUS ACADEMY FOR CIVIL SERVICES


Q: Which of the following peaks/ mountain ranges is/are located within India?
1. Mahabharat range
2. Makalu range
3. Annapurna
4. Dhauladhar range
Select the correct answer using the codes given below.
(a) 1 only
(b) 1 and 3 only
(c) 2, 3 and 4 only
(d) 4 only
Answer: d
Except dhauladhar Range all the give n ranges/peaks are situated in Nepal with Makalu and
Annapurna being part of inner Himalayas and the Mahabharata being part of the Middle Himalayas.

Q: With reference to Karewas, consider the following statements:


1. They are alluvial deposits found at foothills of Himalayas.
2. They are beneficial for cultivation of saffron.
Which of the statements given above is/are correct?
(a) 1 only
(b) 2 only
(c) Both 1 and 2
(d) Neither 1 nor 2

Answer: B
Statement 1 is not correct. Karewas are the thick deposits of glacial clay and other materials
embedded with moraines.
Statement 2 is correct. Karewa formations are useful for the cultivation of Zafran, a local variety of
saffron in the Kashmir Himalayas.

Q: Consider the following pairs:


Mountain Pass State
1. Shipki La : Himachal Pradesh
2. Niti : Sikkim
3. Thang La : Uttarakhand
4. Nathu La : Kashmir
Which of the pairs given above are correctly matched?
(a) 1 and 2 only
(b) 2 and 4 only
(c) 3 and 4 only
(d) 1 and 3 only

Answer: D
The correct arrangement is as follows
1. Shipki La pass Himachal Pradesh
2. Niti Pass Uttarakhand
3. Thang La Uttarakhand

© APTI PLUS ACADEMY FOR CIVIL SERVICES


4. Nathu La Sikkim
Some other passes are as follows:
Bara Lapcha La in Himachal Pradesh, Karakoram pass in Kashmir, Nathu la pass in Sikkim, Bomdi La in
Arunachal Pradesh, etc.

Q: Which of the following is/are present in Himachal and Uttarakhand Himalayas ?


1. Valley of Flowers
2. Dun Formation
3. Nagtibba hills
Select the correct answer using the code given below.
(a) 1 only
(b) 1 and 2 only
(c) 2 and 3 only
(d) 1, 2 and 3

Answer: D
Valley of Flowers is famous national park situated in Uttarakhand Himalayas.
Duns are longitudinal valleys and many such Duns like Dehradun,Nalaagarh Dun, Kalka Dun are found
here.
Lesser Himalayas are called Nagtibba hills in Uttarakhand.

Q: Consider the following peaks:


1. Kanchenjunga
2. Nanga Parbat
3. Nanda Devi
4. Mt Everest
Which of the following is the correct arrangement of above peaks from west to east?
(a) 1-2-4-3
(b) 2-3-4-1
(c) 2-1-3-4
(d) 2-4-3-1
Answer; B
Nanga Parbat is the peak in Kashmir. Nanda Devi is in Uttarakhand, the Mt Everest is located in Nepal
and Kanchenjunga is located in Sikkim.
Arranging them from west to east the order comes as follows
Nanga Parbat ----Nanda Devi------ Mt Everest-----Kanchenjunga

Q: Consider the following statements with regard to eastern and western Himalayas:
1. Eastern Himalayas receive more rainfall than Western Himalayas.
2. Western Himalayas have thick evergreen forests and eastern Himalayas are devoid of it.
3. The Himachal and Shiwalik ranges of western Himalayas merge into single range in eastern Himalayas.
Which of the statements given above is/are correct?
(a) 1 only
(b) 2 and 3 only
(c) 1 and 3 only
(d) 1, 2 and 3

© APTI PLUS ACADEMY FOR CIVIL SERVICES


Answer: C
Statement 1 is correct as Eastern Himalayas receive around 200 cm of rainfall compared to western
Himalayas. As matter of fact we know that North eastern regions receive greater rainfall than western
regions
Statement 2 is incorrect and it's the opposi
te, i.e.; Western Himalayas have thick coniferous forests
while Eastern Himalayas are covered with evergreen forests . It can also be deduced form rainfall and
snowfall patterns.
Statement 3 is correct. In case of western Himalayas, there 3 different ranges of Himadri, Himachal
and Shiwalik. The Himachal and shiwalik ranges merge into single range in eastern Himalayas.

Q: Which of the following statements is/are correct regarding Eastern Hills (Purvanchal ranges)?
1. Its general alignment is from South east to North west.
2. Barak and Lohit rivers originate from these ranges.
3. Loktak Lake is located in this region.
Select the correct answer using the code given below.
(a) 1 and 2 only
(b) 3 only
(c) 2 and 3 only
(d) 1, 2 and 3

Answer: B
Statement 1 is incorrect: General Alignment of Purvanchal ranges are from North to South. These are
Patkai Bum Naga hills , Mizo hills.
Statement 2 is incorrect: Barak, Dhansari, Dhikhu are important rivers which originate from
Purvanchal
range. Lohit originates from Tibet hImalayas
Statement 3 is correct: Loktak lake is located in Manipur which is part of Purvanchal ranges.

Q: Which of the following is the correct description of Rohi?


(a) Arid Plains in Thar Desert Region
(b) Fertile Tracts of Land in the Rajasthan plains
(c) Local name of Luni River in Gujarat
(d) Salt lakes in the Rajasthan plains
Answer: B
Rohi are the fertile tracts of land in the Rajasthan plains composed of Rajasthan Bagar, which is semi-
arid plain in the eastern side of the desert. Cultivation is done in small patches in this area with the help
of water from small streams.

© APTI PLUS ACADEMY FOR CIVIL SERVICES


Q: Which of the following statements is/are correct with regard to Terai region?
1. It is a swampy area formed by reappearance of streams submerged in Bhabhar region.
2. The region is characterised by thicker vegetation than Bhabhar region.
3. They are more wide spread near eastern Himalayas than western Himalayas.
Select the correct answer using the codes given below.
(a) 1 only
(b) 1 and 3 only
(c) 2 and 3 only
(d) 1, 2 and 3

Answer: D
Statement 1 is correct. It is a strange terrain, an area consisting of marshy underground seepage. It is
formed when water from the Bhabhar areas seep down in the soil and suddenly appear, when the flat
plains begin. It thus creates a swampy area.
Stat ement 2 is correct as the region is heavily forested and covered with thick vegetation.
Statement 3 is correct as Terai region is more widespread in the east as compared to the west
because
eastern Himalayas get more rainfall than the Western Himalayas.

Q: Which of the following statements regarding the Northern plains of India is/are correct?
1. It was originally developed as geo- synclinal depression.
2. These plains are formed by alluvial deposits brought by the river systems of Indus and Ganga only.
Select the correct answer using the code given below.
(a) 1 only
(b) 2 only
(c) Both 1 and 2
(d) None of the above

Answer: a

 Statement 1 is correct. Plains are formed by the river Indus, the Ganga and the Brahmaputra.
Originally, it was a geo-synclinal depression which attained its maximum development during
the third phase of the Himalayan mountain formation approximately about 64 million years ago.
 Statement 2 is not correct. The northern plains are formed by the alluvial deposits brought by
the rivers - the Indus, the Ganga and the Brahmaputra.

Q: Syntaxial bend is associated with which of the following geographical features in India?
(a) Northern plains
(b) Himalayas
(c) Deccan Plateau
(d) Central highlands

Answer: b
The general east-west trend of the Himalayas terminates suddenly at its western and eastern
extremities and the ranges are sharply bent southward in deep knee-bend flextures which are called

© APTI PLUS ACADEMY FOR CIVIL SERVICES


syntaxial bends. The western syntaxial bend is near Nanga Parbat where the Indus has cut a deep
gorge. The geological formations here take sharp hairpin bends as if they were bent round pivotal
points obstructing them. There is a similar hair- pin bend in Arunachal Pradesh where the mountains
take a sharp bend from the eastern to southern direction after crossing the Brahmaputra river.

Q: Consider the following statements regarding the Northern Plains.


1. River tends to disappear in Bhabar region due to deposition of heavy rocks and boulders.
2. Swampy and marshy conditions are peculiarity of Tarai region due to non-demarcated channel of
rivers.
3. Riverine islands and sand bars can be found in Brahmaputra plains.
4. Punjab serves as a water divide between Indus and Ganges river system.
Select the correct statement(s) using the codes given below.
a) 1, 2 and 3 only
b) 1, 3 and 4 only
c) 2, 3 and 4 only
d) None of the above
Answer: a

Explantion:
States of Haryana and Delhi form the water divide between Indus and Ganga river system. Hence,
statement 4 is incorrect.

Q: Which among the following are the right bank tributaries of Yamuna?
1. Chambal
2. Betwa
3. Ken
Select the correct answer using the codes given below.
a) 1 and 2 only
b) 2 and 3 only
c) 1 and 3 only
d) 1, 2 and 3

Answer: d
All the above mentioned rivers are right bank tributaries of Yamuna.
Other Right bank tributaries of Yamuna are –Chambal , Sindh , Betwa , Ken Left Bank tributaries are
Hindan , Rind , Sengar , Varuna etc.

Q:Abor, Mishmi and Dafla are :


(a) Names of tribes inhabiting Arunachal Himlayas.
(b) Names of hills present in Arunachal Himlayas.
(c) Names of rivers flowing through Aruncahal Himlayas.
(d) Both (a) and (b) are correct.

Answer: D
Abor, Mishmi, Dafla, Miri are the names of tribes inhabiting in Arunachal ranges. Hills of Lesser
Himalayas present in this region are also named on the name of major tribe inhabiting this hills and thus

© APTI PLUS ACADEMY FOR CIVIL SERVICES


they are also named as Dafla, Miri, Abor,Mishmi

Q:Which of the following statements regarding the Meghalaya plateau is/are correct?
1. It is an extension of the Peninsular plateau.
2. It is rich in of coal, iron ore and uranium.
3. It receives maximum rainfall from Arabian sea branch of South west monsoon.
Select the correct answer using the code given below.
(a) 1 only
(b) 1 and 2 only
(c) 2 and 3 only
(d) 1, 2 and 3

Answer: B
Statement 1 is correct as at the time of Himalayan formation, the north-eastward movement of the
Indian plate caused a huge fault between the Rajmahal hills and the Meghalaya plateau. Thus the north
eastern plateau comprising the Meghalaya plateau and the Karbi-Anglong hills got separated.

Statement 2 is correct as the composition of rocks in the Meghalaya plateau is similar to the
Chhotanagpur plateau. Thus, they are also rich in minerals like coal, iron ore, uranium, silimanite etc.

Statement 3 is not correct as the Meghalaya Plateau receives rainfall from the Bay of bengal branch of
South west monsoon.

Q: Consider the following statements:


1. The Deccan plateau gets low rainfall due to the rain shadow effect of Western ghats.
2. Alignment of Aravalis is one of the reasons for less rainfall in Rajasthan.
Which of the statements given above is/are correct?
(a) 1 only
(b) 2 only
(c) Both 1 and 2
(d) Neither 1 nor 2
Answer: C
Statement 1 is correct. The deccan plateau receives low rainfall when compared to western parts of
western ghats due to the rain shadow effect of western ghats.
Statement 2 is correct because Aravali range is parallel to the direction of southwest mons
oon. Hence
no orographic rainfall.

Q: Which of the following statements regarding the Central Highlands of Peninsular Plateau is/are
correct?
1. It slopes towards south and southeastern directions.
2. It is marked by the presence of metamorphic rocks.
3. Many tributaries of the river Yamuna have their origin in this region.
Select the correct answer using the code given below.
(a) 1 only
(b) 2 and 3 only
(c) 1 and 2 only
(d) 1, 2 and 3

© APTI PLUS ACADEMY FOR CIVIL SERVICES


Answer: b

o Statement 1 is incorrect as the general elevation of the Central Highlands ranges between 700-1,000
m above the mean sea level and it slopes towards the north and northeastern directions.
o Statement 2 is correct as this region has undergone metamorphic processes in its geological history,
which can be corroborated by the presence of metamorphic rocks such as marble, slate, gneiss, etc.
o Statement 3 is correct as most tributaries of the river Yamuna is from Vindhyas and Kaimur ranges

Q: With reference to the Deccan Trap, consider the following statements:


1. It is made of basaltic lava flows.

2. It extends to parts of Madhya Pradesh, Rajasthan and Uttar Pradesh.


3. The region is deficient in ground water resources.
Which of the statements given above is/are correct?
(a) 1 and 2 only
(b) 1 and 3 only
(c) 2 and 3 only
(d) 1, 2 and 3

Answer: d

 Statement 1 is correct: The Deccan Traps are one of the largest volcanic provinces in the world.
It consists of more than 6,500 feet of flat-lying basalt lava flows and covers an area of nearly
200,000 square miles (500,000 square km) in west-central India.
 Statement 2 is correct: In Western and Central India, Deccan trap is exposed mainly in the states
of Maharashtra, Madhya Pradesh, Karnataka, Gujarat, and Andhra Pradesh and also has its
nominal presence in southern parts of Uttar Pradesh and eastern parts of Rajasthan. Based on
the area of occurrence, the Deccan Lavas have been further classified into four classes viz.
Malwa Trap: occurring in Malwa region of Madhya Pradesh, Mandla Traps: occurring in Mandla
region of Madhya Pradesh, Saurashtra Trap: occurring in Saurashtra region of Gujarat and Main
Deccan plateau: occurring in States of Maharashtra, Karnataka and Andhra Pradesh.
 Statement 3 is correct: This region is deficient in ground water due to a 1,200 meter thick
covering of basalt which obstructs percolation.

Q: Which among the following statement is incorrect about the peninsular plateaus?
a) The general elevation of Peninsular plateau is from the east to the west, slopping westwards.
b) Delhi Ridge is the northwest extension of the peninsular plateau.
c) The north-eastern extension of peninsular plateau is separated by Malda fault.
d) Ravines of Madhya Pradesh are relief features of Peninsular plateau.
Answer: a
Explanation:
General elevation of peninsular plateau is from west to east. That is why most rivers in peninsular
India flow towards Bay of Bengal . Hence , statement a is incorrect.
Karbi Anglong and Shillong are North eastern Extension of peninsular plateau .
The Ravines of Chambal , Bhind and Morena are a part of Peninsular plateau.

© APTI PLUS ACADEMY FOR CIVIL SERVICES


Q: Which of the following regions or their parts are classified as the most probable earth quake zones by
the Bureau of Indian Standards?
1. North Bihar
2. Rann of Kutch
3. Andaman and Nicobar islands
4. Kanyakumari-Koodankulam Belt
Select the correct answer using the code given below.
(a) 1, 2 and 3 only
(b) 2 and 4 only
(c) 3 and 4 only
(d) 1, 2, 3 and 4
Answer: A
o Bureau of Indian Standard has grouped the country into four seismic zones viz. Zone-II, Zone-III, Zone-
IV and Zone-V. Of these, Zone V is seismically the most prone region with higher incidence of activities
of earthquakes and Zone II is the least prone region with lower incidents of activities of earthquakes.

o Broadly, Zone-V comprises of entire northeastern India, parts of Jammu and Kashmir, Himachal
Pradesh, Uttaranchal, Rann of Kutch in Gujarat, part of North Bihar and Andaman & Nicobar islands.

o Zone-IV covers remaining parts of Jammu & Kashmir and Himachal Pradesh, Union Territory of Delhi,
Sikkim, northern parts of Uttar Pradesh, Bihar and West Bengal, parts of Gujarat and small portions of
Maharashtra near the west coast and Rajasthan.

o Zone-III comprises of Kerala, Goa, Lakshadweep islands, remaining parts of Uttar Pradesh, Gujarat and
West Bengal, parts of Punjab, Rajasthan, Madhya Pradesh, Bihar, Jharkhand, Chhattisgarh, Maharashtra,
Orissa, parts of Telangana, parts of Andhra Pradesh, Tamilnadu and Karnataka.

o Zone-II covers remaining parts of the country.

Q: Which among the following sets of pair is correctly matched?


a) Mango Shower – Gujrat
b) Blossom Shower – Karnataka
c) Nor westers – Bihar
d) Loo – Bengal

Answer: b

Explanation:
Mango Showers, or ‘mango rains’, is a pre-monsoon rainfall. These rains occur Kerala, Karnataka and
Konkan and Goa. They help in the early ripening of mangoes, and often referred to as 'mango showers.
In Karnataka pre-monsoon rainfalls are called cherry ‘blossoms shower’ due to their effect on the coffee
plantations.
Nor-westers or the Kalbaishakhi is a local thunderstorm which occurs in Bengal.
The Loo is a strong, hot and dry summer afternoon wind from the west which blows over the
western Indo-Gangetic Plain region of North India and Pakistan.

© APTI PLUS ACADEMY FOR CIVIL SERVICES


Q: Which of the following statements regarding the coastal plains of India is/are not correct?
1. The western coastal plains are an example of submerged coastal plain.
2. The eastern coastal plains provides natural conditions for the development of ports and harbors.
3. The western coastal plains are broader than the eastern coastal plains.
Select the correct answer using the code given below.
(a) 1 only
(b) 2 and 3 only
(c) 1 and 3 only
(d) 1, 2 and 3
Answer: B

Explanation
Statement 1 is correct as the western coastal plains are an example of submerged coastal plain.
Statement 2 is not correct because of the submergence of western coastal plains, it is a narrow belt
and provides natural conditions for the development of ports and harbours. Because of emergent
nature of Eastern coastal plains, it has less number of ports and harbours. The continental shelf extends
up to 500 km into the sea, which makes it difficult for the development of good ports and harbours.
Statement 3 is not correct as compared to the western coastal plain, the eastern coastal plain is
broader and is an example of an emergent coast.

Q: Which of the following statements regarding Lakshadweep Islands is/are correct?


1. These islands are divided by the Eleven degree channel.
2. The entire island group is built of volcanic deposits.
3. Minicoy is the largest island in these group of islands.
Select the correct answer using the code given below.
(a) 1 only
(b) 2 and 3 only
(c) 1 and 3 only
(d) 1, 2 and 3

Answer: C
Statement 1 is correct as the Eleven degree channel divides the island into 2 parts with Amindivi
islands in the north and Cannanore in the south.
Statement 2 is incorrect as the entire island group is built of coral deposits and not volcanic deposits.
Statement 3 is correct as the Minicoy is the largest among 36 islands in the group.

Q: Consider the following statements with regard to Kayals:


1. They are the backwaters of Kathiawad coast.
2. They are not suitable for fishing and inland navigation.
Which of the statements given above is/are correct ?
(a) 1 only
(b) 2 only
(c) Both 1 and 2
(d) Neither 1 nor 2

Answer: D
Explanation:

© APTI PLUS ACADEMY FOR CIVIL SERVICES


Statement 1 is not correct. Kayals are a distinguishing feature of Malabar coast (Kerala) and not
Kathiawad coast (Gujarat).
Statement 2 is not correct. Kayals are used for fishing and inland navigation. Every year the famous
Nehru Trophy Vallamkali (boat race) is held in Punnamada Kayal in Kerala.

Q: Majuli, one of the largest riverine island in the world, is located on which river?
(a) Ganga
(b) Manas
(c) Brahmaputra
(d) Padma

Answer: C
Majuli, one of the largest riverine island in the world, is located on the Brahmaputra River, Assam.

Q: Which of the following statements is/are correct regarding the Andaman and Nicobar islands?
1. Saddle peak is the highest point of Andaman and Nicobar islands.
2.The Barren island, India’s only active volcano, is situated here.
3. The nine degree channel separates the Andaman islands in the north and Nicobar islands in the south.
Select the correct answer using the code given below.
(a) 1 and 2 only
(b) 3 only
(c) 2 and 3 only
(d) 1, 2 and 3

Answer: A
Statement 1 is correct. Saddle peak is the highest point of Andaman and Nicobar islands.
Statement 2 is correct as the Barren Island which is situated in the Nicobar islands, is India's only
active volcano.
Statement 3 is not correct it is the ten de
gree channel which separates the Andaman in the north and
Nicobar in the south. Nine degree channel is a channel in the Indian ocean which separates Minicoy
island from Lakshadweep island group.

Q: Sir Creek a disputed region between India and Pakistan is:


a) a coral island
b) a volcanic island
c) a tidal estuary
d) a backwater lagoon

Answer: c

 Sir Creek is a 96 km (60 mi) tidal estuary on the border of India and Pakistan. The creek, which
opens up into the Arabian Sea, divides the Gujarat state of India from the Sindh province of
Pakistan.

© APTI PLUS ACADEMY FOR CIVIL SERVICES


Q: Which among the following statement is incorrect regarding the Indian Deserts?
a) Rivers in this region are ephemeral.
b) The underlying rock structure is an extension of peninsular plateau.
c) Longitudinal dunes are the features of this region.
d) There are evidences that the region has always been an arid one and desert.

Answer: d.

Explanation:
It is believed that during Mesozoic era, north west of Aravali was under the sea. Which can be
corroborated by the evidence available at wood fossil park at Aakal and marine deposits around
Brahmsar near Jaisalmer. Hence , statement d is incorrect. Indian desert is also part of the peninsular
plateau.

Q: West flowing rivers of peninsular India do not form deltas but form only estuaries.
Which of the following reasons can be attributed to this phenomenon?
1. Passage through hard rocks.
2. Non-perennial flow of water.
3. Less rainfall in western part of peninsular India
Select the correct answer using the codes given below.
(a) 1 only
(b) 1 and 2 only
(c) 2 and 3 only
(d) 1, 2 and 3
Answer: A
Explanation:
 Statement 1 is correct. The west flowing rivers of India do not form delta because they passes
through hard rocks. This means that these are devoid of sediments which are essential for
formation of delta.
 Statement 2 is not correct. Non-perennial flow of water is characteristic of Peninsular rivers,
including the east flowing rivers which form deltas.
 Statement 3 is not correct: Western part of the peninsular India receives higher rainfall because
of the presence of Western ghats. Statement 3 is not a reason.

Q: With reference to the Peninsular rivers, consider the following statements:


1. They are known to change their course frequently.
2. They are characterized by trellis and radial drainage patterns.
3. They have larger catchment area than the Himalayan rivers.
Which of the statements given above is/are correct?
(a) 1 and 2 only
(b) 2 only
(c) 3 only
(d) 1, 2 and 3

Answer: b

© APTI PLUS ACADEMY FOR CIVIL SERVICES


Explanation:
 Statement 1 is not correct. Peninsular rivers have smaller and fixed course with well-adjusted
valleys, while the Himalayan rivers have long course, flowing through the rugged mountains
experiencing headward erosion and river capturing. In plains the Himalayan rivers are known for
meandering and shifting of course.
 Statement 2 is correct. Peninsular rivers are characterised by trellis, radial and rectangular
drainage patterns while Himalayan rivers are characterised by dendritic pattern in plains.
 Statement 3 is not correct. Himalayan rivers have larger catchment area than the peninsular
rivers.

Q: Why Jhelum forms meanders in its youth stage, a feature associated with the mature stage of the
rivers?
(a) Due to low gradient of flow in its youth stage reducing its Kinetic Energy.
(b) Due to large number of pebble rocks present in its path significantly reducing its speed.
(c) Due to local base level provided by the erstwhile larger lake.
(d) Due to large number of dams constructed reducing the volume of water.

Answer: C
Explanation
Rivers generally form meanders in its mature stage but Jhelum, flowing through the Kashmir valley,
forms meanders in its youth stage.
About 2 million years ago when the Kashmir valley, surrounded by Great Himalayan Range in north-east
and Pir Panjal Range in southeast, was submerged under water. Due to tectonic upliftment of Pir Panjal
range, drainage of the region was impounded and a big lake of about 5000 sq km area was formed.
Because of endogenic forces, Baramullah Gorge was created and this vast lake was drained through this
gorge leaving behind sediments and these deposits are called Karewas. ("Karewa" in Kashmiri means
"elevated table-land River Jhelum: So, meanders are formed by Jhelum river due to these local base
levels of Karewas, provided by that formerly existing large lake which drained leaving behind finer
sediments. It gets silt and sediments to deposit when it is in its upper course or youth stage.

Q: Consider the following statements with regard to tributaries of Indus:


1. Chenab flows through the Wular lake.
2. Satluj originates near Mansarovar in Tibet and passes through Shipki la pass.
3. Beas river flows through the states of Himachal pradesh, Punjab and Jammu and Kashmir.
Which of the statements given above is/are not correct?
(a) 1 and 2 only
(b) 2 and 3 only
(c) 1 and 3 only
(d) 1, 2 and 3

Answer: C
 Statement 1 is incorrect: The Jhelum, an important tributary of the Indus, rises from a spring at
Verinag situated at the foot of the Pir Panjal in the south-eastern part of the valley of Kashmir. It
flows through Srinagar and the Wular lake before entering Pakistan through a deep narrow
gorge.
 Statement 2 is correct: The Satluj originates in the Rakas lake near Mansarovar at an altitude of
4,555 m in Tibet where it is known as Langchen Khambab. It flows almost parallel to the Indus

© APTI PLUS ACADEMY FOR CIVIL SERVICES


for about 400 km before entering India, and comes out of a gorge at Rupar. It passes through
the Shipki La on the Himalayan ranges and enters the Punjab plains.
 Statement 3 is incorrect: Beas river originates in Himachal pradesh and enters the Punjab plains
where it meets the Satluj near Harike. Beas does not flow through Jammu and Kashmir.

Q: Which of the following rivers originate from Aravalli Hills?


1. Luni
2. Sabarmati
3. Mahi
Select the correct answer using the code given below.
(a) 1 only
(b) 1 and 2 only
(c) 2 and 3 only
(d) 1, 2 and 3

Answer: B
Luni and Sabarmati rivers originate from the Aravallis. Mahi, on the other hand, originates from
Vindhyachal Hills. Hence, (b) is the correct answer.

Q: Consider the following statements regarding Dendritic Pattern of drainage:


1. The drainage pattern resembling the branches of a tree is known as dendritic.
2. The rivers originating from the Amarkantak range present a good example of it.
Which of the statements given above is/are correct?
(a) 1 only
(b) 2 only
(c) Both 1 and 2
(d) Neither 1 nor 2

Answer: A
 Statement 1 is correct. The drainage pattern resembling the branches of a tree is known as
dendritic. The examples of which are the rivers of northern plain.
 Statement 2 is not correct. The rivers originating from the Amarkantak range present a good
example of radial pattern.

Q: With reference to river Krishna, consider the following statements:


1. It acts as a geographical border between Telangana and Andhra Pradesh.
2. It originates from Trimbak in Nasik.
3. Rivers Koyna and Tungabhadra are its important tributaries.
Which of the following statements given above is/are correct?
(a) 1 only
(b) 1 and 3 only
(c) 2 and 3 only
(d) 1, 2 and 3

Answer: B
 Statement 1 is correct. River Krishna acts as a geographical divide between Telangana and
Andhra Pradesh.

© APTI PLUS ACADEMY FOR CIVIL SERVICES


 Statement 2 is incorrect. River Krishna has its origin in the Western Ghats just to the north of
Mahabaleshwar. River Godavari rises from Trimbak.
 Statement 3 is correct. The Koyna, Ghatprabha, Malprabha, Bhima and Tungabhadra are its
important tributaries.

Q: Which of the following rivers passes through Mizoram?


1. Dihang
2. Barak
3. Kaladan
4. Subansiri
Select the correct answer using the code given below.
(a) 1 and 2 only
(b) 2 and 3 only
(c) 1, 3 and 4 only
(d) 1, 2, 3 and 4

Answer: B
o There are a number of rivers which flow through the state. Some of the rivers which are majorly part
of the country of Bangladesh but passes through the state of Mizoram also are Thega River and the
Surma- Meghna river system (Barak River).
o Tiau River, Tlawng, Barak, Kaladan, Karnaphuli are some major rivers of Mizoram.

Q: The Ganga river flows through which of the following states?


1. Himachal Pradesh
2. Chattisgarh
3. Bihar
4. Odisha
Select the correct answer using the code given below.
(a) 1 and 3 only
(b) 2, 3 and 4 only
(c) 3 only
(d) 1, 2, 3 and 4

Answer: C
The Ganga river is shared by :
Uttaranchal (110 km)
Uttar Pradesh (1,450 km),
Bihar (445
km)
West Bengal (520 km).
However it touches the border of Jharkhand as well.

Q: The river originates from the Bokhar Chu glacier. In tibet it is known as Singi Khamban or Lion's
mouth. It forms spectacular gorge near Gilgit in Jammu and Kashmir.
Which of the following rivers is described above?
(a) Chenab
(b) Jhelum
(c) Satluj
(d) Indus

© APTI PLUS ACADEMY FOR CIVIL SERVICES


Answer: D
Explanation
The Indus also known as the Sindhu, is the westernmost of the Himalayan rivers in India. It originates
from a glacier near Bokhar Chu (31°15' N latitude and 81°40'E longitude) in the Tibetan region at an
altitude of 4,164 m in the Kailash Mountain range. In Tibet, it is known as Singi Khamban; or Lion mouth.
After flowing in the northwest direction between the Ladakh and Zaskar ranges, it passes through
Ladakh and Baltistan. It cuts across the Ladakh range, forming a spectacular gorge near Gilgit in Jammu
and Kashmir.

Q: Which among the following rivers drain into the Arabian Sea?
1. Subarnarekha
2. Periyar
3. Baharatpuzha
Select the correct answer using the code given below.
(a) 1 and 3 only
(b) 2 only
(c) 2 and 3 only
(d) 1, 2 and 3

Answer: C
Subarnarekha River flows through the states of Jharkhand, West Bengal and Odisha. It drains into Bay
of Bengal.

The Periyar is the second largest river of Kerala. It drains into Arabian Sea.

The longest river of Kerala, Bharathapuzha rises near Annamalai hills. It is also known as Ponnani. It
drains into Arabian Sea.

Q: Which of the following is/ are not the tributaries of Brahmaputra?


1. Lohit
2. Barak
3. Dhansiri
4. Tista
Select the correct answer using the codes given below.
(a) 1 and 2 only
(b) 2 only
(c) 2, 3 and 4 only
(d) 3 and 4 only

Answer: B
Barak river is part of Surma Meghna river system and originates in Manipur hills and flows through
Manipur, Mizoram and Assam.
Dihang, Dibang, Dhansiri, Tista, Lohit and Manas are the tributaries of Brahmputra in India

© APTI PLUS ACADEMY FOR CIVIL SERVICES


Q: Consider the following pairs:
Place Bank of river
e) Hampi : Godavari
f) Ujjain : Shipra
g) Puri : Mahanadi
Which of the pairs given above is/are correctly matched?
(a) 1 only
(b) 3 only
(c) 2 and 3 only
(d) 1, 2 and 3

Answer: c

 Shipra is a river in Madhya Pradesh state of central India. The river rises in the ujjaini situated in
Indore district, and flows north across the Malwa Plateau to join the Chambal River at the MP-
Rajasthan boundary in Mandsaur district. It is one of the sacred rivers in Hinduism. The holy city of
Ujjain is situated on its east bank.
 The Mahanadi is one of the largest Indian peninsular rivers that drains into the Bay of Bengal. The
857 km long river originates in Raipur district of the central Indian state of Madhya Pradesh and
flows through the eastern state of Orissa before meeting the sea. The famous city of Puri is located
on its banks.
 Tungabhadra is a major river in the south Indian peninsula. Hampi is located on the south bank of
this river.

Q 7: With reference to the tributaries of Ganga, consider the following statements:


1. Both Alaknanda and Bhagirathi originate from Satopanth glacier.
2. Kali Ganga joins Alaknanda at Karna Prayag.
3. Son is the longest left bank tributary of Ganga
Which of the statements given above is/are correct?
(a) 1 and 2 only
(b) 2 and 3 only
(c) 1, 2 and 3
(d) None of the above

Answer: d
 Statement 1 is not correct: The Ganga is the most important river of India both from the point
of view of its basin and cultural significance. It rises in the Gangotri glacier near Gaumukh
(3,900 m) in the Uttarkashi district of Uttarakhand. Here, it is known as the Bhagirathi. The
Alaknanda has its source in the Satopanth glacier above Badrinath.
 Statement 2 is not correct: The Alaknanda consists of the Dhauli and the Vishnu Ganga which
meet at Joshimath or Vishnu Prayag. The other tributaries of Alaknanda such as the Pindar joins
it at Karna Prayag while Mandakini or Kali Ganga meets it at Rudra Prayag.
 Statement 3 is not correct: The Son is Ganga's major right bank tributary. The important left
bank tributaries are the Ramganga, the Gomati, the Ghaghara, the Gandak, the Kosi and the
Mahananda.

© APTI PLUS ACADEMY FOR CIVIL SERVICES


Q: Arrange the following rivers in decreasing order of their catchment areas in India.
1. Ganga
2. Indus
3. Brahmaputra
4. Kaveri
Select the correct answer using the code given below.
(a) 1-3-2-4
(b) 1-2-3-4
(c) 3-1-2-4
(d) 2-1-3-4

Answer: b
 The correct sequence is: Ganga- Indus- Brahmaputra-Kaveri. Their respective catchment areas in
India are: Ganga: 86,452 sq.km, Indus: 321,289 sq.km, Brahmaputra: 194,413 sq.km, Kaveri:
81,155 sq.km

Q: Which of the following sub-branches of the Arabian sea branch of southwest monsoon merges with
the Bay of Bengal branch?
1. Branch crossing Western Ghats and passing through Narmada and Tapi valley.
2. Branch traveling parallel to the Aravallis.
Select the correct answer using the code given below.
(a) 1 only
(b) 2 only
(c) Both 1 and 2
(d) Neither 1 nor 2

Answer: C
Explanation:
Statement 1 is correct. This branch after crossing Narmada and Tapi valleys causes rainfall in
Chotanagpur Plateau. After that, it enters the Ganga plains and merges with the Bay of bengal branch.
Statement 2 is correct. This branch merges with bay of bengal branch near Punjab and Haryana plains
and then move towards Western Himalayas.

Q: Which among the following is/are the effects of tropical cyclones brought by easterly jet streams into
India during summer season?
1. They affect the distribution of monsoon rainfall in India.
2. They reduce the amount of monsoon rainfall in India.
Select the correct answer using the code below:
(a) 1 only
(b) 2 only
(c) Both 1 and 2
(d) Neither 1 nor 2

Answer: A

© APTI PLUS ACADEMY FOR CIVIL SERVICES


Explanation
Statement 1 is correct. The easterly jet stream steers the tropical depressions into India. These
depressions play a significant role in the distribution of monsoon rainfall over the Indian subcontinent.
The tracks of these depressions are the areas of highest rainfall in India. The frequency at which these
depressions visit India, their direction and intensity, all go a long way in determining the rainfall pattern
during the southwest monsoon period.

Statement 2 is not correct. They do not reduce the amount of monsoon rainfall in India. The tracks of
these depressions are the areas of highest rainfall in India.

Q: With reference to Western Disturbances, consider the following statements:


1. They originate over the Mediterranean Sea.
2. They enter India under the influence of the westerly jet streams.
3. Their arrival is marked by an increase in prevailing night temperatures.
Which of the statements given above is/are correct?
(a) 1 and 2 only
(b) 1 only
(c) 2 and 3 only
(d) 1, 2 and 3

Answer: d
Statement 1 is correct. The western cyclonic disturbances which enter the Indian subcontinent from
the west and the northwest during the winter months, originate over the Mediterranean Sea.
Statement 2 is correct. They are brought into India by theesterly
w jet stream.
Statement 3 is correct. An increase in the prevailing night temperature generally indicates an advance
in the arrival of these cyclones disturbances.

Q: The physiography or relief of a place affects which of the following phenomena?


1. Temperature
2. Direction of the wind
3. Speed of the wind
4. Amount of rainfall
Select the correct answer using the code given below.
(a) 1, 2 and 3 only
(b) 1, 2 and 4 only
(c) 3 and 4 only
(d) 1, 2, 3 and 4

Answer: D
All the give phenomena are influenced by relief of a place. The physiography or relief affects the
temperature, air pressure, direction and speed of wind and the amount and distribution of rainfall. For
example. The windward sides of Western Ghats and Assam receive high rainfall during June-September
whereas the southern plateau remains dry due to its leeward situation along the Western Ghats.

© APTI PLUS ACADEMY FOR CIVIL SERVICES


Q: Consider the following statements with regards to variability of rainfall in India.
1. There is high rainfall as well as high rainfall variability in Western Ghats.
2. There is low rainfall, but high rainfall variability in Rajasthan.
Which of the statements given above is/are correct?
(a) 1 only
(b) 2 only
(c) Both 1 and 2
(d) Neither 1 nor 2

Answer: B
Statement 1 is Incorrect. A var iability of less than 25 per cent exists on the western coasts, Western
Ghats, northeastern peninsula, eastern plains of Ganga, northeastern India, Uttaranchal and Himachal
Pradesh and south-western part of Jammu and Kashmir. These areas have an annual rainfall of over 100
cm.
Statement 2 is Correct. A variability of over 50 per cent exists in the western part of Rajasthan,
northern part of Jammu and Kashmir and interior parts of the Deccan plateau. These areas have an
annual rainfall of less than 50 cm. Rest of India have a variability of 25-50 per cent and these areas
receive an annual rainfall between 50 -100 cm.

Q: With reference to Koeppen's scheme of climate classification, consider the following pairs:
Type of Climate: Associated Region
1. As : West Coast of India
2. E : Jammu and Kashmir
3. Aw : Extreme western Rajasthan
4. Cwg : Gangetic Plains
Which of the pairs given above is/are correctly matched?
(a) 1, 2 and 3 only
(b) 1, 3 and 4 only
(c) 2 and 4 only
(d) 1, 2, 3 and 4

Answer: C
Type of Climate as per Koeppen's scheme of climate classification:
o Amw (Monsoon with short dry season)- West coast of India south of Goa
o As (Monsoon with dry summer) - Coromandel coast of Tamil Nadu
o Aw (Tropical savannah) - Most of the Peninsular plateaus, south of the Tropic of Cancer
o Bwhw (Semi-arid steppe climate) - North-western Gujarat, some parts of western Rajasthan and
Punjab
o Bwhw (Hot desert) - Extreme western Rajasthan
o Cwg (Monsoon with dry winter) - Ganga plain, eastern Rajasthan, northern Madhya Pradesh, most of
North-east India
o Dfc (Cold humid winter with short summer) - Arunachal Pradesh
o E (Polar type) - Jammu and Kashmir, Himachal Pradesh and Uttaranchal

© APTI PLUS ACADEMY FOR CIVIL SERVICES


Q: With reference to climate in India, consider the following statements:
1. In Andaman and Nicobar Islands the annual and daily range of temperature is very small.
2. In the Thar desert the daily range of temperature is large.
Which of the statements given above is/are correct?
(a) 1 only
(b) 2 only
(c) Both 1 and 2
(d) Neither 1 nor 2

Answer: C
o Statement 1 is correct. The tropical areas being close to the equator experiences high temperature
throughout the year with small daily and annual range. Andaman and Nicobar Islands fall in this zone.
o Statement 2 is correct. North West India experiences very hot summers and equally harsh winters. In
fact in a single day the temperature varies a great deal. For example, in the Thar desert, if the day
temperature is around 50 degree celcius, it might drop to 15-20 degree at night.

Q: Which of the following pairs is/are correctly matched?


Local wind Beneficial for
1. Kalbaisakhi: Jute cultivation
2. Loo: Wheat cultivation
3. Cherry blossoms: Grape cultivation
4. Mango showers: Mango cultivation
Select the correct answer using the code given below.
(a) 1 and 4 only
(b) 2 and 4 only
(c) 3 and 4 only
(d) 1, 2, 3 and 4

Answer: A
Explanation
Kalbaisakhi They bring rainfall in West Bengal, Assam and Orissa before monsoon season and are
beneficial for jute and rice cultivation.
Loo they are strong and dusty summer winds in north India. They are not beneficial for any crop
cultivation.
Mang o showers - They cause rainfall along Karnataka coast during pre-monsoon period and hence are
beneficial for early ripening of mangoes.
Loo they are strong and dusty summer winds in north India. They are not beneficial for any crop
cultivation.
Cherry blossom cause rainfall along Kerela and Karnataka coast during pre-monsoon period and
hence are beneficial for Coffee cultivation in this area.

Q: Consider the following statements in reference to Retreating monsoon season:


1. The monsoon starts retreating from eastern India.
2. The weather is marked by clear skies.
3. The eastern peninsula receives heavy rainfall.
Which of the statements given above is/are correct?
(a) 1 and 3 only
(b) 2 and 3 only

© APTI PLUS ACADEMY FOR CIVIL SERVICES


(c) 1 only
(d) 1, 2 and 3

Answer: B
 Statement 1 is Incorrect. The months of October and November are known for retreating monsoons.
By the end of September, the southwest monsoon becomes weak as the low pressure trough of the
Ganga plain starts moving southward in response to the southward march of the sun. The monsoon
retreats from the western Rajasthan by the first week of September.
 Statement 2 is Correct. The retreating southwest monsoon season is marked by clear skies and rise in
temperature. The land is still moist. Owing to the conditions of high temperature and humidity, the
weather becomes rather oppressive. This is commonly known as the ‘October heat’.
Statement 3 is Correct . The weather in the retreating monsoon is dry in north India but it is
associated with rain in the eastern part of the Peninsula. Here, October and November are the rainiest
months of the year.

Q: Which one of the following phenomenon happens in India when the sun shines vertically over the
Tropic of Capricorn?
(a) High pressure develops over Northwestern India.
(b) Low pressure develops over Ganga plain.
(c) Peninsular India under the influence of south west trade winds.
(d) Easterly jet stream over Himalayas

Answer: A
By the end of December (22nd December), the sun shines vertically over the Tropic of Capricorn in
the southern hemisphere. The weather in this season is characterised by feeble high pressure conditions
over the northern plain.

Q: With reference to Western Disturbances, consider the following statements:


1. They originate over the Mediterranean Sea.
2. They enter India under the influence of the westerly jet streams.
3. Their arrival is marked by an increase in prevailing night temperatures.
Which of the statements given above is/are correct?
(a) 1 and 2 only
(b) 1 only
(c) 2 and 3 only
(d) 1, 2 and 3

Answer: d
Statement 1 is correct. The western cyclonic disturbances which enter the Indian subcontinent from
the west and the northwest during the winter months, originate over the Mediterranean Sea.
Statement 2 is correct. They are brought into India by the westerly jet stream.
Statement 3 is correct. An increase in the prevailing night temperature generally indicates an advance
in the arrival of these cyclones disturbances.

© APTI PLUS ACADEMY FOR CIVIL SERVICES


Q: With reference to Koeppen's scheme of climate classification, consider the following pairs:
Type of Climate: Associated Region
1. As : West Coast of India
2. E : Jammu and Kashmir
3. Aw : Extreme western Rajasthan
4. Cwg : Gangetic Plains
Which of the pairs given above is/are correctly matched?
(a) 1, 2 and 3 only
(b) 1, 3 and 4 only
(c) 2 and 4 only
(d) 1, 2, 3 and 4

Answer: C
Type of Climate as per Koeppen's scheme of climate classification:
o Amw (Monsoon with short dry season)- West coast of India south of Goa
o As (Monsoon with dry summer) - Coromandel coast of Tamil Nadu
o Aw (Tropical savannah) - Most of the Peninsular plateaus, south of the Tropic of Cancer
o Bwhw (Semi-arid steppe climate) - North-western Gujarat, some parts of western Rajasthan and
Punjab
o Bwhw (Hot desert) - Extreme western Rajasthan
o Cwg (Monsoon with dry winter) - Ganga plain, eastern Rajasthan, northern Madhya Pradesh, most of
North-east India
o Dfc (Cold humid winter with short summer) - Arunachal Pradesh
o E (Polar type) - Jammu and Kashmir, Himachal Pradesh and Uttaranchal

Q: Which of the following areas receive annual rainfall of more than 200 cm?
1. Western slopes of Western Ghats
2. Western Himalayas
3. Northern Tamil Nadu
Select the correct answer using the code given below.
(a) 1 only
(b) 1 and 3 only
(c) 2 and 3 only
(d) 1, 2 and 3

Answer: a
Areas receiving more than 200cm of rainfall annually are :
Western slopes of western ghats, Meghalaya hills, southern slopes of Eastern himalayas, Assam,
Arunachal pradesh.
Western Himalayas are arid and receive less than 100 cm of rainfall.
Northe rn Tamil Nadu receives a rainfall of nearly 50-100 cm annually.

Q: With reference to Inter Tropical Convergence Zone (ITCZ), consider the following statements:
1. It is a high pressure zone located at the equator.
2. Upward shift of ITCZ in summer season causes onset of southwest monsoon.
3. Onset of north east monsoon is independent of shifting of ITCZ.
Which of the statements given above is/are correct?
(a) 1 and 2 only

© APTI PLUS ACADEMY FOR CIVIL SERVICES


(b) 2 only
(c) 1 and 3 only
(d) 1, 2 and 3

Answer: B
Statement 1 is not correct. ITCZ is a low pressure zone located at the equator where trade winds
converge and so it is a zone where air tends to ascend.
Statement 2 is correct. In July, the ITCZ is located around 20°N
-25°N latitudes (over the Gangetic
plain), sometimes called the monsoon trough. This monsoon trough encourages the development of
thermal low over north and northwest India. Due to the shift of ITCZ, the trade winds of the southern
hemisphere cross the equator between 40° and 60°E longitudes and start blowing from southwest to
northeast due to the Coriolis force. It becomes southwest monsoon.
Statement 3 is not correct. In winter, the ITCZ moves southward, and so the reversal of winds from
northeast to south and southwest, takes place. They are called northeast monsoons.

Q: Which of the following factors affect Climate of India?


1. La Nina
2. Westerly jet streams
3. Tibetan plateau
4. Indian ocean
Select the correct answer using the code given below.
(a) 1 and 2 only
(b) 2, 3 and 4 only
(c) 1, 2 and 3 only
(d) 1, 2, 3 and 4

Answer: D
La Nina - Occurrence of La Nina and El Nino affects the South west monsoon and rainfall pattern
across
India.
Westerly jet streams - They are prevalent over north India during winters and are also responsible for
winter rains in North west region.

Tibetan plateau - Tibetan plateau is the main reason for change of pressure system across India and it
influences the occurrence of monsoon as well.
Indian Ocean - India is surrounded by Indian ocean, bay of bengal and arabian sea. This acts as the
major source of moisture for south west and north east monsoon.

Q: Which of the following is/are associated with retreating monsoon in India ?


1. Low pressure over Ganga plains
2. Cyclonic rainfall over Tamil Nadu coast
3. October heat
Select the correct answer using the code given below.
(a) 1 and 2 only
(b) 1 only
(c) 2 and 3 only
(d) 1, 2 and 3

© APTI PLUS ACADEMY FOR CIVIL SERVICES


Answer: C
Statement 1 is not correct. The south -west monsoon starts retreating from northern india in early
October. Hence, the months of October and November are known for retreating monsoon. The reason
for this retreat is that the monsoon trough of low pressure over the Ganga plains becomes weaker due
to the apparent movement of the sun. The low pressure trough is gradually replaced by high pressure.
Statement 2 is correct. The low pressure conditions from Ganga plains are transferred to the centre of
Bay of Bengal by early November. This shift of low pressure area is marked by cyclonic depressions
which originate over the Andaman Sea. Some of the cyclonic depressions manage to cross the eastern
coasts of southern peninsula resulting in heavy and widespread rains on the coast of Tamil Nadu and
Orissa.
Statement 3 is correct. The retreat of monsoon is marked by clear skies and drop in night
temperature.
The land remains moist. The combination of high temperature and humidity gives rise to an oppressive
weather. This is commonly known as 'October heat'

Q: Which of the following statements are correct regarding Indian Monsoon?


1. The Bay of Bengal branch is deflected towards India by Arakan Hills in Myanmmar.
2. Coromandal coast receives substantial rainfall from Bay of Bengal branch of Monsoon.
Select the correct answer using the code given below.
(a) 1 only
(b) 2 only
(c) Both 1 and 2
(d) Neither 1 nor 2

Answer: A
Statement 1 is correct. Bay of bengal branch is deflected towards India from Arakan hills of Mayanmar
which are part of Purvanchal ranges. Great Himalayas are only found till diphu pass of Arunachal
pradesh.
Statement 2 is not correct. Coromandal cost i.e. Tamilnadu coast is parallel to Bay of bengal branch of
monsoon, hence it does not receive rainfall from it. It receives rainfall from the north east monsoon.

Q: Consider the following pairs:


Tree Forest
1. Neem : Tropical Thorn
2. Teak : Tropical Evergreen
3. Rosewood : Tropical Deciduous
Which of the pairs given above is/are correctly matched?
(a) 1 only
(b) 1 and 2 only
(c) 2 and 3 only
(d) None

Answer: a
Explanation:
Tropical Thorn Forests: Important species found are babool, ber, and wild date palm, khair, neem,
khejri, palas, etc. Tropical thorn forests occur in the areas which receive rainfall less than 50 cm.
Tropical Evergreen and Semi Evergreen Forests: Speci es found in these forests include rosewood,

© APTI PLUS ACADEMY FOR CIVIL SERVICES


mahogony, aini, ebony, etc. These forests are found in the western slope of the Western Ghats, hills of
the northeastern region and the Andaman and Nicobar Islands.
Tropical Deciduous Forests: These are the mos t widespread forests in India. They are also called the
monsoon forests. Teak, sal, shisham, hurra, mahua, amla, semul, kusum, and and salwood etc. are the
main species of these forests.

Q: Mangrove forests are present in which of the following river deltas?


1. Godavari
2. Krishna
3. Mahanadi
Select the correct answer using the code given below.
(a) 1 only
(b) 1 and 2 only
(c) 2 and 3 only
(d) 1, 2 and 3

Answer: d
Mangroves forest comes under littoral and swamp forests. They support saline resistant vegetation
like sundari tree. They are found in the deltas of: Ganga-brahamaputra, Mahanadi, Godavari and Krishna
etc. Apart from these deltas, they are found in Andaman and Nicobar and Gujarat.

Q: Which among the following are found in the state of Tamil Nadu?
1. Tropical dry evergreen Forests.
2. Tropical deciduous Forests.
3. Littoral and swamp forests.
4. Montane forests.
Select the correct answer using the code given below.
(a) 1 and 2 only
(b) 1, 2 and 3 only
(c) 3 and 4 only
(d) 1, 2, 3 and 4

Answer: D
All the given four vegetation types are found in Tamil Nadu:
1. Tropical Evergreen Forests
2. Tropical Deciduous Forests
3. Littoral and Swamp forests
4. Montane forests

Q: Which of following are the characteristic features of Tropical Evergreen forests of India?
1. They are multilayered forests with dense canopy.
2. Trees reach great heights.
3. Trees shed their leaves during specific season.
Select the correct answer using the code given below.
(a) 1 only

© APTI PLUS ACADEMY FOR CIVIL SERVICES


(b) 1 and 2 only
(c) 2 and 3 only
(d) 1, 2 and 3

Answer: B
Statement 1 is Correct. They are multilayered forests with dense canopy. They also have a dense layer
of herbs and grasses, however due to dense canopy they do not grow much.
Statement 2 is Correct. They reach great heights of over 60m.
Statement s3 Incorrect.
i They do not have fixed season to shed their leaves, to flower or fruition.
That is why they appear to be green all-year round.

Q: According to National Commission on Agriculture, Social Forestry includes which of the following?
1. Urban Forestry
2. Rural Forestry
3. Farm Forestry
Select the correct answer using the code given below.
(a) 1 and 3 only
(b) 2 only
(c) 2 and 3 only
(d) 1, 2 and 3

Answer: D

Explanation:
The National Commission on Agriculture (1976) has classified social forestry into three categories.
These are Urban forestry, Rural forestry and Farm forestry.
Urban forestry pertains to the raising and management of trees on public and privately owned lands in
and around urban centres such as green belts, parks, roadside avenues, industrial and commercial green
belts, etc.
Rural forestry lays emphasis on promotion of agro-forestry and community-forestry.
Farm forestry is a term applied to the process under which farmers grow trees for commercial and non-
commercial purposes on their farm lands.
Hence, (d) is the correct answer.

Q: Shola forests are found in which of the following regions?


1. Nilgiri Hills
2. Annamalai hills
3. Andaman and Nicobar Islands
Select the correct answer using the code given below.
(a) 1 only
(b) 1 and 2 only
(c) 2 and 3 only
(d) 1, 2 and 3

Answer: B
Explanation:
At 1,500 m above the sea level, vegetation is temperate in the higher regions, and subtropical on the
lower regions of the Western Ghats, specially in Kerala, Tamil Nadu and Karnataka. The temperate

© APTI PLUS ACADEMY FOR CIVIL SERVICES


forests are called Sholas in the Nilgiris, Anaimalai and Palani hills. Some of the other trees of this forest
of economic significance include, magnolia, laurel, cinchona and wattle.
They are not found in Andaman and Nicobar Islands where Tropical evergreen forests and mangroves
are found.

Q: Which among the following trees are example of Tropical Deciduous vegetation?
1. Mulberry
2. Sisham
3. Rosewood
4. Sandalwood
Select the correct answer using the code given below.
(a) 1, 2 and 4 only
(b) 1 and 2 only
(c) 1 and 3 only
(d) 2, 3 and 4 only

Answer: A
Types of trees found in Moist Decidous forests: sal, teak, shisham, sandalwood, mahua, mulberry,
palas, semul etc.
Types of tree s found in Dry Decidous forests: sal, teak, tendu, palas, amaltas, khair etc.

: Consider the following statements regarding the characteristics of red soil:


1. It's soil profile is generally shallow and is acidic in nature.
2. It is rich in nitrogen and organic matter, thus suitable for pulses.
Which of the statements given above is/are correct?
(a) 1 only
(b) 2 only
(c) 1 and 2 Both
(d) None of the above

Answer: d
Statement 1 is not correct. Red soils are generally shallow but have pH values between 6.6 to 8.0, so
they are slightly alkaline.
Statement 2 is not correct as the red soils are poor in nitrogen, potash, phosphorous and organic
matter.

Q: Which of the following statements is/are correct regarding Black soil?


1. It is poor in nitrogen, phosphorous and organic matter.
2. It has a fine texture and is clayey in nature.
3. It is rich in lime, iron and magnesium.
Select the correct answer using the code given below:
(a) 1 only
(b) 1 and 3 only
(c) 2 and 3 only
(d) 1, 2 and 3

© APTI PLUS ACADEMY FOR CIVIL SERVICES


Answer: d
Explanation:
Statement 1 is correct as the Black soil is poor in nitrogen, phosphorus and organic matter.
Statement 2 is not correct as Black soil is fine textured and clayey in nature.
Statement 3 is correct as the lava soil is rich in lime, iron and magnesiumince
s it has been formed by
denudation of volcanic rocks

Q: Which of the following statements is/are correct regarding the characteristics of alluvial soil?
1. It is confined only to the northern plains of India.
2. It is deficient in humus except the Ganga delta which is rich in humus.
3. The alluvial soils are more alkaline in drier areas.
Select the correct answer using the code given below.
(a) 1 only
(b) 2 only
(c) 2 and 3 only
(d) 1, 2 and 3

Answer: C
Statement 1 is not correct as the alluvial soils are found in all the river plains across India including
coastal plains.
Statement 2 is correct as the alluvial soil is deficient in humus except the Ganga delta which is rich in
humus.
Statement 3 correct
is as the alluvial soils are alkaline in drier areas.

Q: Which of the following statements regarding laterite soil is/are correct?


1. They are mainly found in the areas with high temperature and high precipitation.
2. They do not retain moisture and are suitable for crops of tea, coffee and cashewnuts.
Select the correct answer using the code given below:
(a) 1 only
(b) 2 only
(c) Both 1 and 2
(d) Neither 1 and 2

Answer: C
Statement 1 is correct. Laterite soild develop in the areas of high temperature and high precipitation.
Statement 2 is correct as laterite soils are coarse textured and can't retain moisture. Apart from this
they are also devoid of most of minerals and humus, so they are not fertile. Thus only special crops like
tea, coffee , tapioca, cashewnuts etc are grown.

Q: Which of the following can help preventing soil erosion?


1. Use of chemical fertilizers
2. Terrace Farming
3. Shifting cultivation
Select the correct answer using the codes given below.
(a) 1 and 3 only
(b) 1 and 2 only
(c) 2 only
(d) 1, 2 and 3

© APTI PLUS ACADEMY FOR CIVIL SERVICES


Answer: C
Chemical fertilizers in the absence of organic manures are also harmful to the soil. Unless the soil gets
enough humus, chemicals harden it and reduce its fertility in the long run.
Terrace Farming and Regulated Forestry are popular methods of soil conservation. Contour bunding,
Contour terracing, regulated forestry, controlled grazing, cover cropping, mixed farming and crop
rotation are some of the remedial measures which are often adopted to reduce soil erosion.
Shifting cultivation causes soil erosion.

Q: Which of the following statements is/are correct regarding red soils?


1. Red soil forms the second largest soil group after the alluvial soil.
2. It is red in colour because of the presence of copper oxides.
3. It looks yellow when it occurs in a hydrated form.
Select the correct answer using the code given below.
(a) 1 only
(b) 1 and 3 only
(c) 2 and 3 only
(d) 1, 2 and 3

Answer: B
o Statement 1 is correct because red soil is found all over the peninsular plateau, forming a circle around
the black soil regions.
o Statement 2 is not correct because red soil gets its red colour from iron oxides.
o Statement 3 is correct because the red soil looks yellowish on being wet due to hydration of the salts
present in it.

Q: Consider the following statements:


1. Self-ploughing is a phenomenon associated with it.
2. They are formed by the denudation of volcanic rocks.
3. They are deep and impermeable
Which of the following is the type of soil that above statements attribute to ?
(a) Black Soil
(b) Laterite Soil
(c) Peat Soil
(d) Arid Soil

Answer: A
 The black soil is very deep. These soils are also known as the Regur Soil or the Black Cotton Soil. The
black soils are generally clayey, deep and impermeable. They swell and become sticky when wet and
shrink when dried. So, during the dry season, these soil develop wide cracks. Thus, there occurs a kind of
self-ploughing.
Because of this character of slow absorption and loss of moisture, the black soil retains the moisture
for a very long time, which helps the crops, especially, the rain fed ones, to sustain even during the dry
season.
Black soils are formed by the denudation of volcanic rocks. Chemically, the black soils are rich in lime,
iron, magnesia and alumina. They also contain potash. But they lack in phosphorous, nitrogen and
organic matter. The colour of the soil ranges from deep black to grey.

© APTI PLUS ACADEMY FOR CIVIL SERVICES


Q: With reference to “Western Ghats”, which of the following statement(s) is/are correct?
1. It is also known as Sahyadri.
2. It is a UNESCOs World Heritage Site.
Select the correct answer using the code given below:
(a) 1 only
(b) 2 only
(c) Both 1 and 2
(d) Neither 1 and 2

Answer: c
Explanation:
Western Ghats also known as Sahyadri (Benevolent Mountains) is a mountain range that runs parallel to
the western coast of the India peninsula, located entirely in India. It is a UNESCO World Heritage
Site and is one of the eight “hottest hot-spots” of biological diversity in the world
The area is one of the world’s ten Hottest biodiversity hotspots and it is well known for its rich and
unique flora and fauna.

Q: With reference to “Nilgiri tahr”, which of the following statement(s) is/are correct?
1. It is the state animal of Kerala, locally known as the Nilgiri ibex or ibex.
2. The Nilgiri tahr is fully protected (Schedule I) by the Indian Wildlife (Protection) Act of 1972.
3. It is listed under critically endangered category in IUCN Red list.
Select the correct answer using the code given below.
(a) 1 and 2 only
(b) 2 Only
(c) 1 and 3 only
(d) 1, 2 and 3

Answer: b
Statement 1 is incorrect: The Nilgiri tahr known locally as the Nilgiri ibex or simply ibex, is an ungulate
that is endemic to the Nilgiri Hills and the southern portion of the Western Ghats in the states of Tamil
Nadu and Kerala in Southern India. It is the state animal of Tamil Nadu, locally known as the Nilgiri
ibex or ibex.
Statement 2 is correct: The Nilgiri tahr is fully protected (Schedule I) by the Indian Wildlife (Protection)
Act of 1972.
Statement 3 is incorrect: It is listed under endangered category in IUCN Red list.

Q: Consider the following statements:


1. Natural growth of population does not take into account the migration of population.
2. Actual growth of population is always greater than the natural growth of population.
Which of the statements given above is/are correct?
(a) 1 only
(b) 2 only
(c) Both 1 and 2
(d) Neither 1 nor 2
Answer: A
Statement 1 is correct. Natural Growth = Births - Deaths
Actual Growth of Population = {Births - Deaths} + {In Migration - Out Migration}

© APTI PLUS ACADEMY FOR CIVIL SERVICES


Net migration will determine whether natural growth s greater
i or actual growth is greater. Therefore
it cannot be said that actual growth of population is always greater than the natural growth of
population.
Hence, statement 2 is incorrect.

Q: Consider the following pairs:


Type of settlement pattern Associated Region
1. Linear Pattern: Plain areas or wide inter montane valleys
2. Rectangular Pattern: Regions along a river or railway line
3. Circular Pattern: Region around Lakes or tanks
Which of the pairs given above is/are not correctly matched?
(a) 1 and 2 only
(b) 2 only
(c) 3 only
(d) 1, 2 and 3

Answer: A
Linear pattern: In such settlements houses are located along a road, railway line, river, canal edge of a
valley or along a levee.
Rectangular pattern: Such patterns of rural settlements are found in plain areas or wide inter
montane valleys. The roads are rectangular and cut each other at right angles.
Circular pattern: Circular villages develop around lakes, tanks and sometimes the village is planned in
such a way that the central part remains open and is used for keeping the animals to protect them from
wild animals.
Therefore, pairs 1 and 2 are incorrectly matched.

Q: Which of the following migration streams is the most dominant one in India?
(a) Rural to rural
(b) Rural to urban
(c) Urban to rural
(d) Urban to urban

Answer: A
Among the four migration streams, the rural to rural migration is the most dominant one which
majorly consists of female migrants who migrate after marriage.

Q: Which of the following are the 'Push factors' of Migration?


1. Poor living conditions
2. Political turmoil
3. Better job opportunities
4. Epidemics
Select the correct answer using the code given below.
(a) 1 and 4 only
(b) 2 and 3 only
(c) 1, 2 and 4 only
(d) 1, 2, 3 and 4

© APTI PLUS ACADEMY FOR CIVIL SERVICES


Answer: C
There are two sets of factors that influence migration.
The Push factors make the place of origin seem less attractive for reasons like unemployment, poor
living conditions, political turmoil, unpleasant climate, natural disasters, epidemics and socio-economic
backwardness.
The Pull factors make the placef odestination seem more attractive than the place of origin for
reasons like better job opportunities and living conditions, peace and stability, security of life and
property and pleasant climate.
Therefore, (c) is the correct answer.

Q: Consider the following pairs:


Type of Rural Settlement: Associated region
1. Clustered Settlement: Meghalaya region
2. Hamleted Settlement: Lower Ganga plain
3. Dispered Settlement: Bundelkhand region
Which of the pairs given above is/are correctly matched?
(a) 1 and 2 only
(b) 1 and 3 only
(c) 2 only
(d) 1, 2 and 3

Answer: C
Clustered Settlements is a compact or closely built up area of houses. In this type of village the general
living area is distinct and separated from the surrounding farms, barns and pastures. Such settlements
are generally found in fertile alluvial plains and in the northeastern states. Sometimes, people live in
compact village for security or defence reasons, such as in the Bundelkhand region of central India and
in Nagaland. In Rajasthan, scarcity of water has necessitated compact settlement for maximum
utilisation of available water resources.
Hamleted Settlements is fragmented into several units physically separated from each other bearing a
common name. Such villages are more frequently found in the middle and lower Ganga plain,
Chhattisgarh and lower valleys of the Himalayas.
Dispersed or isolated settlement pattern in India appears in the form of isolated huts or hamlets of few
huts in remote jungles, or on small hills with farms or pasture on the slopes. Many areas of Meghalaya,
Uttaranchal, Himachal Pradesh and Kerala have this type of settlement.

Q. What is demographic transition-


(a) The population of a country turning young
(b) The population of any region changes from rural agrarian to urban industrial society
(c) Country attaining demographic dividend with more than two third of its total population being young
(d) The population of a country turning old

Answer: b
Demographic transition theory can be used to describe and predict the future population of any area.
The theory tells us that population of any region changes from high births and high deaths to low births
and low deaths as society progresses from rural agrarian and illiterate to urban industrial and literate
society. These changes occur in stages which are collectively known as the demographic cycle.
Hence, Option-b should be the answer

© APTI PLUS ACADEMY FOR CIVIL SERVICES


Q: With reference to the model of Demographic Transition Theory (DTT), arrange the following in
correct sequence of their occurrence?
1. High birth rate, low death rate
2. High birth rate, High death rate
3. Low birth rate, low death rate
Select the correct answer using the code given below.
(a) 1-2-3
(b) 2-1-3
(c) 3-1-2
(d) 2-3-1

Answer: B
Demographic transition theory tells us that population of any region changes from high births and
high deaths to low births and low deaths as society progresses from rural agrarian and illiterate to urban
industrial and literate society.
The first stag e has high birth rate and high death rate because people reproduce more to compensate
for the deaths due to epidemics and variable food supply.
Fertility remains high in the beginning of second stage but it declines with time. This is accompanied
by reduced mortality rate. Improvements in sanitation and health conditions lead to decline in mortality.
In the last stage, both fertility and mortality decline considerably. The population is either stable or
grows slowly.
Therefore,-1-3 2 is the correct sequence.

Q: Which of the following statements is/are correct regarding Demographic structure?


1. High proportion of youth population would mean that the region has high birth rate.
2. Triangular shaped population pyramid with wide base is typical of developing countries.
3. Bell shaped population pyramid tapering to top represents almost equal birth and death rate.
Consider the correct code from following options:
(a) 1 only
(b) 1 and 2 only
(c) 2 and 3 only
(d) 1, 2 and 3

Answer: D
Statement 1 is correct as countries with more youth population like India has more birth rate.
A population pyramid is used to show age -sex structure of population. Statement 2 is correct as
triangular shaped pyramid with a wide base and is typical of less developed countries. These have larger
populations in lower age.
Statement 3 is correct. When age -sex pyramid is bell shaped and tapers towards top then birth and
death rate is almost equal tending to constant population.

Q: Consider the following statements about the Demographic Transition Theory:


1. The growth of population is slow in the first stage.
2. Population is stable in the second stage.
3. In the last stage, population becomes urbanized, literate and deliberately controls the family size.
Which if the above statements is/are not correct?
(a) 1 and 3 only

© APTI PLUS ACADEMY FOR CIVIL SERVICES


(b) 2 only
(c) 2 and 3 only
(d) 1, 2 and 3

Answer: B
Statement 1 is correct. In the first stage of demographic transition, the population goes through a low
growth because of high mortality leading to high death rate.
Statement 2 is not correct. Population is stable in the third stage and not the second stage. Fertility
remains high in the beginning of second stage but it declines with time. Improvements in sanitation and
health conditions lead to decline in mortality. Because of this gap the net addition to population is high.
Statement 3 is correct. In the last stage, both fertility and mortality decline considerably. The
population is either stable or grows slowly. The population becomes urbanised, literate and has high
technical knowhow and deliberately controls the family size.

Q: Consider the following pairs:


Age sex pyramid Representative Population
1. Triangular Pyramid with flat base : Expanding population
2. Bell shaped pyramid : Declining population
3. Narrow base with tapered top : Constant population
Which of the pairs given above is/are correctly matched?
(a) 1 only
(b) 2 only
(c) 1 and 2 only
(d) 1, 2 and 3

Answer: A
Expanding Populations:
The age-sex pyramid of Nigeria for example is a triangular shaped pyramid with a wide base and is
typical of less developed countries. These have larger populations in lower age groups due to high birth
rates.
Ex: Bangladesh and Mexico etc.
Constant Population:
In the case of Australia age-sex pyramid is bell shaped and tapered towards the top. This shows birth
and death rates are almost equal leading to a near constant population.
Declining Populations:
The Japan pyramid has a narrow base and a tapered top showing low birth and death rates. The
population growth in developed countries is usually zero or negative.

Q: The ‘Pemberton Line’ and the ‘Johnstone Line’ are related to the boundary between India and
a. Srilanka
b. Pakistan
c. Myanmar
d. Bangladesh

Answer: c
In the medieval ages, Manipur and Burmese kings often wrested the valley from each other until the
British defeated the Burmese and signed the Treaty of Yandaboo in 1826. But the valley was returned to
Burma in the second treaty and a boundary line between British India and Burma was drawn by Captain

© APTI PLUS ACADEMY FOR CIVIL SERVICES


R.B. Pemberton. The Pemberton Line had left out certain restive Kuki areas that were included in a
rectified boundary in 1881 called Johnstone Line. The boundary was redrawn again in 1896 to have 38
pillars and be known as Maxwell or Pemberton-Johnstone-Maxwell Line.

Q: Consider the following statements regarding The World Customs Organisation (WCO)
1. Was established as an associate body to the World Trade Organisation.
2. Has divided its membership into six regions, with India presently being one of the regional
heads.
3. Represents 193 customs administrations across the globe that collectively process
approximately 98% of world trade.
Which of the statements given above is/are correct?
(a) 1 and 3 only
(b) 2 only
(c) 2 and 3 only
(d) 1, 2 and 3

Answer: c
 The WCO, established in 1952 as the Customs Co-operation Council (CCC) is an independent
intergovernmental body whose mission is to enhance the effectiveness and efficiency of
customs administrations.
 The CCC was intially a Customs Committee set up by a group of a few European Governments
based on the principles of the GATT. The name ‘WCO’ was adopted in 1994 to reflect its status
as a truly global intergovernmental institution.
 Today, the WCO represents 182 Customs administrations across the globe that collectively
process approximately 98% of world trade.
 India’s Central Board of Indirect Taxes and Customs (CBIC) is the nodal body for the WCO; India
recently became the vice-chair (regional head) of the Asia-Pacific region.

Q: Under the Bharatmala Pariyojana programme,


1. Five new national corridors will be built.
2. Over 500 districts in the country will be linked by national highways.
3. Feeder routes, economic corridors, and border roads will be constructed.
Select the correct answer using the code given below:
(a) 1 and 3 only
(b) 2 only
(c) 2 and 3 only
(d) 1, 2 and 3

Answer: c
Fifty new national corridors will be built; the programme will help connect 550 districts in the country
through NH linkages; National corridors, feeder routes, economic corridors, coastal roads, greenfield
expressways and border roads will be constructed. Deadline for the first phase of the scheme is 2022.

© APTI PLUS ACADEMY FOR CIVIL SERVICES


Q: Consider the following pairs:
Southern Cape – Located in country
1. Cape of Good Hope – South Africa
2. Cape Horn – Argentina
3. Cape Leeuwin – New Zealand
Which of the pairs given above is/are correctly matched?
(a) 1 only
(b) 2 only
(c) 2 and 3 only
(d) 1, 2 and 3

Solution: a.
 Cape Horn refers to the steep rocky headland on Hornos Island, Tierra del Fuego Archipelago in
southern Chile. It is located off the southern tip of mainland South America.
 Cape Leeuwin is the most south-westerly mainland point of the Australian continent, in the
state of Western Australia.

Q: Consider the following statements:


1. Under the Blue Revolution Scheme, central financial assistance for deep sea fishing can be availed.
2. Of the total fish production in India, the larger share is registered from the inland fisheries sector.
Which of the statements given above is/are correct?
(a) 1 only
(b) 2 only
(c) Both 1 and 2
(d) None of the above

Answer: c
Assistance for Deep-Sea Fishing’ under ‘Blue-Revolution’:
 Under this, the traditional fishermen and their societies / associations can avail financial
assistance to the extent of Rs. 40 lakh per vessel for purchase of deep sea fishing vessel
including equipments to increase fish production.
 Fish production in India is estimated at 11.4 million tonnes, out of which 68% is registered from
inland fisheries sector and the remaining 32% from marine sector. It is expected that the
indigenous fish requirement by 2020 would be 15 million tonnes as against the production of
11.4 million tonnes. This gap of 3.62 million tonnes is expected to be made up by Inland
Aquaculture and also through mariculture.

Q.Arrange the following regions in descending order of their population density:


1. Eastern India
2. North-eastern India
3. Southern India
4. Central India
Select the correct answers using the codes given below:
(a) 1, 2, 3, and 4
(b) 1, 4, 3, and 2
(c) 3, 1, 4, and 2
(d) 4, 1, 3, and 2

© APTI PLUS ACADEMY FOR CIVIL SERVICES


Answer: b
Eastern region has population density of 625 persons per sq. km.
Central region has population density of 417 persons per sq. km.
Southern region has population density of 397 persons per sq. km.
North-eastern region has population density of 176 persons per sq. km

For more information Read: https://www.geographyandyou.com/population/education/population-


density-terms-geography-india/

Q. Consider the following statements:


1. Sex Ratio, in some countries is calculated as the number of males per thousand females.
2. Sex Ratio, in India is calculated as the number of females per thousand males.
Which of the above statements are NOT correct?
(a) only 1
(b) only 2
(c) Both 1 and 2
(d) Neither 1 nor 2

Answer: d
The number of women and men in a country is an important demographic characteristic.
The ratio between the number of women and men in the population is called the Sex Ratio.
In some countries it is calculated as the number of males per thousand females.
In India, the sex ratio is worked out as the number of females per thousand males.
The sex ratio is an important information about the status of women in a country.
Hence, none of the statements are incorrect.

Q. Consider the following statements-


1. In India, a settlement is classified as urban, if it has a municipality, Cantonment Board or
Notified Area Council.
2. Urban population is not engaged in agricultural pursuits, but only in secondary and tertiary sector.
Which of the given statements are true-
(a) only 1
(b) only 2
(c) Both 1 and 2
(d) Neither 1 nor 2.

Answer: a
The census of India, 1991 defines urban settlements as“All places which have municipality, corporation,
cantonment board or notified town area committee and have a minimum population of 5000 persons,
at least 75 per cent of male workers are engaged in non-agricultural pursuits and a density of population
of at least 400 persons per square kilometers are urban.

© APTI PLUS ACADEMY FOR CIVIL SERVICES


Q: Consider the following statements regarding the population density of India as per the recent census
1. Bihar is the most densely populated state followed by Kerala
2. Arunachal Pradesh is least densely populated state in India.
Choose the correct statement/s using the codes given below.
a. 1 only
b. 2 Only
c. Both 1 and 2
d. None of the above
Answer: b
Statement 1 not correct: Bihar is the most densely populated state (1102) followed by west Bengal
(1029). Kerala had density of 859.
Statement 2 correct:

Q: Consider the following regarding the sex ratio in India


1. India witnessed constantly reducing sex ratio since independence.
2. As per the census 2011 Tamil Nadu had highest sex ratio in India
Choose the correct statement/s using the codes given below.
a. 1 only
b. 2 Only
c. Both 1 and 2
d. None of the above

Answer: d
Statement 1 wrong: there is fluctuation in the sex ratio in India since independence.
Statement 2 wrong: Kerala had highest sex ratio of 1048.

Q: Which of the following components are used in calculating Human Development Index (HDI) by
UNDP?
1. Life expectancy at birth
2. Mean years of schooling
3. Expected years of schooling
4. Gross Domestic Product per capita
Select the correct answer using the code given below.
(a) 1 and 2 only
(b) 2 and 3 only
(c) 1, 2 and 3 only
(d) 1,2, 3 and 4 only

© APTI PLUS ACADEMY FOR CIVIL SERVICES


Answer: C
The Human Development Index (HDI) by UNDP uses the following components:
Life expectancy at birth
Mean of years of schooling for adults aged 25 years
Expected years of schooling for children of school entering age
GNI per capita (PPP$)

Q. With reference to approaches to Human development, consider the following pairs:


Approach: Description
1. Welfare Approach: This approach looks at human beings as beneficiaries or targets of all development
activities.
2. Basic Needs Approach : This approach emphasizes on human choices and is linked to income.
3. Capability Approach : This approach is associated with building human capabilities in the areas of
health, education and access to resources.
Which of the pairs given above is/are correctly matched?
(a) 1 and 3 only
(b) 2 only
(c) 3 only
(d) 1, 2 and 3

Answer: A
Approaches to Human Development:

 Income Approach: This is one of the oldest approaches to human development. Human development
is seen as being linked to income. The idea is that the level of income reflects the level of freedom an
individual enjoys. Higher the level of income, the higher is the level of human development.

Welfare Approach: This approach looks at human beings as beneficiaries or targets of all
development activities. The approach argues for higher government expenditure on education, health,
social secondary and amenities. People are not participants in development but only passive recipients.
The government is responsible for increasing levels of human development by maximising expenditure
on welfare.

 Basic Needs Approach: This approach was initially proposed by the International Labour Organisation
(ILO). Six basic needs i.e.: health, education, food, water supply, sanitation, and housing were identified.
The question of human choices is ignored and the emphasis is on the provision of basic needs of defined
sections.
 Capability Approach: This approach is associated with Prof. Amartya Sen. Building human capabilities
in the areas of health, education and access to resources is the key to increasing human development.

© APTI PLUS ACADEMY FOR CIVIL SERVICES


Q: Consider the following statements regarding multi dimensional poverty Index
1. It is developed in 2010 by the Oxford Poverty & Human Development Initiative (OPHI).
2. Standard of living is considered as one of the dimensions of multi dimensional poverty Index.
Choose the correct statement/s using the codes given below.
e. 1 only
f. 2 Only
g. Both 1 and 2
h. None of the above
Answer: c

Statement 1 correct: The Global Multidimensional Poverty Index (MPI) was developed in 2010 by
the Oxford Poverty & Human Development Initiative (OPHI) and the United Nations Development
Program. It replaced the previous Human Poverty Index. The global MPI is released annually by OPHI
and the results published on its website.

Statement 2 correct: The index uses the same three dimensions as the Human Development Index:
health, education, and standard of living. These are measured using ten indicators.

Q: Who among the following economists introduced the term Human Development?
a. Mehbub Al Haque
b. Amartya Sen
c. Adam Smith
d. Both a and b
Answer: d
Explanation: In 1990 two economists - Prof. Mehbub Al Haque and Prof. Amartya Sen introduced the
concept of Human Development. From 1990 onwards, United Nations Development Programme
(UNDP), each year calculate Human Development Index (HDI) and publish as a report which is known as
Human Development Report (HDR).

© APTI PLUS ACADEMY FOR CIVIL SERVICES


Q:. Consider the following statements:
1. The first population census in India was conducted in 1872 but its first complete census was
conducted only in 1891.
2. India has a highly even pattern of population distribution.

Which of the statement(s) given above is/are correct?


(a)Only 1
(b) Only 2
(c) Both 1 and 2
(d) Neither 1 nor 2

Answer (d)
The first population census in India was conducted in 1872 but its first complete census was conducted
only in 1881. Therefore, statement 1 is not correct.
India has a highly uneven pattern of population distribution. Therefore, statement 2 is also not correct.
Hence, the correct answer is option (d)

Q: He was the first nationalist leader who sought close contact with the masses and was in this respect a
forerunner of Gandhiji. He was the first Congress leader to suffer several terms of imprisonments for the
sake of the country. During the initial stages of the First World War, he urged people to cooperate with
the British Government. He authored the books ‘The Arctic Home of the Vedas’ and ‘Gita Rahasya’.
These statements refer to
a. Madan Lal Dhingra
b. Bal Gangadhar Tilak
c. Savarkar
d. Pherozeshah Mehta

Answer: b

Q: Consider the following statements:


1. As per the National Family Health Survey-4, more than one-third children are underweight.
2. As per the NFHS-4, more than one-third children are stunted.
3. The Constitution declares that improving the nutrition of the country’s people is one of the
primary duties of the government.
Which of the statements given above is/are correct?
(a) 1 and 3 only
(b) 2 only
(c) 3 only
(d) 1, 2 and 3
Answer: d
 In 2015-16, according to National Family Health Survey-4: 38.4 per cent of India’s children,
below the age of five, are stunted, while 35.7 per cent are underweight.
 Article 47 of the Constitution states that, “the State shall regard raising the level of nutrition and
standard of living of its people and improvement in public health among its primary duties”.

© APTI PLUS ACADEMY FOR CIVIL SERVICES


Q: Which of the following types of agriculture is/are noticeable in tropical monsoonal climate?
1. Wet paddy cultivation
2. Lowland cash crops
3. Highland plantation crops
Select the correct answer using the code given below.
(a) 1 only
(b) 1 and 2 only
(c) 2 and 3 only
(d) 1, 2 and 3

Answer: D
1 is correct as rice is the most characteristic crop of monsoonal land.
2 is correct as wide range of cash crops are cultivated in the lowland tropical region for export like
sugarcane, jute etc.
3 is correct as colonization gave rise to cultivation
of certain plantation crops. Tropical upland forests
were cleared for tea and coffee plantation.

Q: Which of the following statements is/are correct with regard to Shifting Cultivation?
1. The ashes left by burning of vegetation add to the fertility of the soil.
2. Mechanized tools like tractors are used in patches for cultivation
3. It is primarily prevalent in the temperate regions of the world.
Select the correct answer using the codes given below.
(a) 1 only
(b) 1 and 2 only
(c) 2 and 3 only
(d) 1, 2 and 3

Answer: A
Shifting agriculture is a form of agriculture, in which an area of ground is cleared of vegetation by
burning and cultivated for a few years and then abandoned for a new area until its fertility has been
naturally restored.
Statemen t 1 correct. The ashes after burning of vegetation increase the fertility of soil for temporary
period.
Statement 2 incorrect. It is a primitive form of agriculture in which primitive tools like sticks and hoes
are used.
Statement 3 incorrect. It isrgely
la prevalent in tropical regions. Locally it is known by different names
such as: Jhuming in NE India, Jadang in Indonesia and Malaysia, Milpa in Central America and Mexico.

Q: The proportion of which of the following land use categories have increased since independence?
1. Forest
2. Area under non agricultural use
3. Barren and waste land
Select the correct answer using the code given below.
(a) 1 only
(b) 1 and 3 only
(c) 2 and 3 only
(d) 1 and 2 only

© APTI PLUS ACADEMY FOR CIVIL SERVICES


Answer: D
The proportion of forest and area under non agricultural use have increased while that of barren and
waste land category has decreased.

Q: The distinct crop seasons - kharif, rabi and zaid- does not exist in southern parts of the country.
Which of the following is primarily responsible for this?
(a) Conducive climatic conditions all through the year.
(b) Greater than 90 per cent irrigation cover in all the states.
(c) Well-targeted government procurement programmes.
(d) Well established networks of Mega Food Parks.

Answer: A
There are three distinct crop seasons in the northern and interior parts of country, namely kharif, rabi
and zaid. However, this type of distinction in the cropping season does not exist in southern parts of the
country. Here, the temperature is high enough to grow tropical crops during any period in the year
provided the soil moisture is available. Therefore, in this region same crops can be grown thrice in an
agricultural year provided there is sufficient soil moisture. Therefore, (a) is the correct answer.

Q: Consider the following statements regarding Zaid cropping season:


1. It is a short duration season beginning after cultivation of kharif crops.
2. It is characterized by cultivation of vegetables and fodder crops.
Which of the statements given above is/are correct?
(a) 1 only
(b) 2 only
(c) Both 1 and 2
(d) Neither 1 nor 2
Answer: B
Statement 1 is not correct. Zaid is a short duration summer cropping season after harvesting of rabi
crops.
Statement 2 is correct. The cultivation of waterme lons, cucumbers, vegetables and fodder crops
during this season is done on irrigated lands.

Q: Which of the following names of shifting agriculture and the region they are practiced in is/are
correctly matched?
1. Jhum- Manipur
2. Pamlou- Mizoram
3. Dipa- Chhattisgarh
(a) 1 only
(b) 1 and 2
(c) 3 only
(d) 2 and 3
Answer: c
Jhumming - Assam, Meghalaya, Mizoram and Nagaland
Pamlou - Manipur
Dipa - Chhattisgarh and Andaman and Nicobar Island.

© APTI PLUS ACADEMY FOR CIVIL SERVICES


Q. Which of the following conditions is/are favorable to tea plantation?
1. High rainfall
2. Warm temperature
3. Well drained soil
4. Sloping land
Select the correct answer using the code given below.
(a) 1, 2 and 3 only
(b) 1, 3 and 4 only
(c) 3 and 4 only
(d) 1, 2, 3 and 4
Answer: d
Climatic conditions for growing tea:
Temperature: 21°C to 29°C is ideal for the production of tea. High temperature is required in summer.
The lowest temperature for the growth of tea is 16°C.
Rainfall: 150-250 cm of rainfall is required for tea cultivation.
Soil: Tea shrubs require fertile mountain soil mixed with lime and iron. The soil should be rich in humus.
Land: Tea cultivation needs well drained land. Stagnation of water is not good for tea plants. Heavy
rainfall but no stagnancy of water, such mountain slopes are good for tea cultivation.

Q: Consider the following statements about Commercial Livestock Rearing:


1. It is highly organized and capital intensive.
2. Rearing of animals is done on scientific basis.
3. It is essentially associated with Least Developed Countries (LDCs).
Which of the following statements given above is/are correct?
(a) 1 only
(b) 1 and 2 only
(c) 2 and 3 only
(d) 1, 2 and 3
Answer: B
Commercial Livestock Rearing is rearing of animals in large ranches organized on scientific basis. It is
highly capital intensive as it uses latest sophisticated technology for processing of meat, hided wool etc.
Therefore, both the statements 1 and 2 are correct.
Commercial livestock ranching is essentially associated with western cultures and is practiced on
permanent ranches. New Zealand, Australia, Argentina, Uruguay and United States of America are
important countries where commercial livestock rearing is practiced. Therefore, statement 3 is not
correct.

Q: Which of the following is not a characteristic feature of Plantation Agriculture?


(a) Large estates or plantations
(b) Single crop specialization
(c) Large capital investment
(d) All of the above are characteristic features of plantation agriculture
Answer: D
The characteristic features of Plantation agriculture are large estates or plantations, large capital
investment, managerial and technical support, scientific methods of cultivation, single crop
specialisation, cheap labour, and a good system of transportation which links the estates to the
factories and markets for the export of the products. Therefore, (d) is the correct answer.

© APTI PLUS ACADEMY FOR CIVIL SERVICES


Q: Consider the following statements regarding Tank Irrigation:
1. It is mainly practiced in Deccan Plateau region.
2. It requires hard rock in the area which do not suck up water.
Which of the statements given above is/are correct?
(a) 1 only
(b) 2 only
(c) Both 1 and 2
(d) Neither 1 nor 2
Answer: C
Tank refers to a small bund of earth or stones built across a stream to impound water which is then
led through narrow channels to the cultivated fields. Statement 1 is correct. It is mainly practiced
Deccan
Plateau region.
Statement 2 is correct. It requires hard rock in the area which do not suck up water. This is theeason
r
why tank irrigation is not found in alluvial plains.

Q: Which of the following statements is/are correct with regard to cotton cultivation in India?
1. It is a tropical crop grown in Kharif season.
2. It requires clear sky during flowering stage.
3. Per hectare output is higher in north west region than black soil region of south.
Select the correct answer using the code given below.
(a) 1 only
(b) 2 and 3 only
(c) 1 and 3 only
(d) 1, 2 and 3
Answer: D
Cotton is a tropical crop grown in kharif season in semi-arid areas of the country. Cotton requires
clear sky during flowering stage. Therefore, statements 1 and 2 are correct.
Statement 3 is correct. Per hectare output of cotton is high under irrigated conditions in north -
western region of the country. Its yield is low in Maharashtra where it is grown under rainfed conditions.

Q: Arabica, Robusta and Liberica varieties are associated with which of the following crops?
(a) Tea
(b) Pulses
(c) Coffee
(d) Oilseeds
Answer: c
There are thre e varieties of coffee i.e. arabica, robusta and liberica. India mostly grows superior
quality
coffee, arabica, which is in great demand in International market.

Q: Consider the following statements regarding North Indian Sugar Industry and Peninsular Indian Sugar
Industry:
1. The yield per hectare is lower in North Indian Sugar Industry than Peninsular Indian Sugar Industry.
2. The production period is longer in North Indian Sugar Industry than Peninsular Indian Sugar Industry.
Which of the statements given above is/are correct?
(a) 1 only
(b) 2 only

© APTI PLUS ACADEMY FOR CIVIL SERVICES


(c) Both 1 and 2
(d) Neither 1 nor 2
Answer: A
In recent decades there has been a tendency of the sugar industry's growth towards the South. This is
because:
In South India the favourable maritime mate cli free from the effects of summer loo and winter frost is
best suited for growing superior varieties of cane.There is longer production period. Therefore,
statement 2 is not correct.
The yield per hectare is lower in North Indian Sugar Industry thaneninsular
P Indian Sugar Industry.
Therefore, statement 1 is correct.

Q: Which of the following varieties of silk accounts for majority of the total natural silk produced in
India?
(a) Mulberry
(b) Eri
(c) Tasar
(d) Muga
Answer: A
Mulberry account s for about 90 per cent of the total natural silk produced in India.
Therefore, (a) is the correct answer.

Q: Which of the following minerals are classified as Non-metallic minerals?


1. Mica
2. Limestone
3. Phosphate
Select the correct answer using the code given below.
(a) 1 and 2 only
(b) 2 only
(c) 1 and 3 only
(d) 1, 2 and 3
Answer: D
On the basis of chemical and physical properties, minerals may be grouped under two main
categories of metallics and non-metallics.
Among the non -metallic minerals produced in India, mica is the important one. The other minerals
extracted for local consumption are limestone, dolomite and phosphate. Therefore, (d) is the correct
answer.

Q: Which of the following oil refineries is/are not located along the coast?
1. Barauni
2. Tatipaka
3. Numaligarh
Select the correct answer using the code given below.
(a) 1 only
(b) 2 and 3 only
(c) 1 and 3 only
(d) 1, 2 and 3
Answer: C

© APTI PLUS ACADEMY FOR CIVIL SERVICES


Barauni refinery is located inland in the state of Bihar. Numaligarh refinery is located in
Assam state.
Tatipaka refinery is located along the coast of Andhra pradesh in Godavari delta.

Q: Khetri, Alwar and Bhilwara are regions associated with which of the following minerals?
(a) Bauxite
(b) Copper
(c) Iron Ore
(d) Coal
Answer: B
All theiveng regions are associated with copper mining. Therefore, (b) is the correct answer.

Q: In India, systematic surveying, prospecting and exploration for minerals is undertaken by which of the
following agencies?
1. Geological Survey of India
2. Oil and Natural Gas Commission
3. National Mineral Development Corporation
Select the correct answer using the code given below.
(a) 1 only
(b) 1 and 2 only
(c) 2 and 3 only
(d) 1, 2 and 3
Answer: D
In India, systematic surveying, prospecting and explorationorf minerals is undertaken by the
Geological
Survey of India (GSI), Oil and Natural Gas Commission (ONGC), Mineral Exploration Corporation Ltd.
(MECL), National Mineral Development Corporation (NMDC), Indian Bureau of Mines (IBM), Bharat
Gold Mines Ltd. (BGML), Hindustan Copper Ltd. (HCL), National Aluminium Company Ltd. (NALCO)
and the Departments of Mining and Geology in various states.

Q: Consider the following pairs:


Energy project Associated Location
1. Nuclear Energy : Kota
2. Wind Energy : Kuchchh
3. Geothermal Energy: Manikaran
Which of the pairs given above is/are correctly matched?
(a) 1 only
(b) 1 and 2 only
(c) 2 and 3 only
(d) 1, 2 and 3
Answer: D
All the given pairs are correctly matched. Kota in Rajasthan has nuclear energy plant. Windower
p
plant
Is at Lamba in Gujarat in Kachchh. There is a geothermal energy plant at Manikaran in
Himachal Pradesh.

© APTI PLUS ACADEMY FOR CIVIL SERVICES


Q: Which of the following is/are the drawbacks of coal found in India?
1. Low calorific value
2. High ash content
3. Majority of the reserves are lignite type
Select the correct answer using the code given below.
(a) 1 only
(b) 1 and 2 only
(c) 2 and 3 only
(d) 1, 2 and 3
Answer: B
The major drawbacks of India coal are low calorific value and high ash content. Majority of coal
reserves are of Bituminous type not Lgnite types.

Q: Which of the following pairs of Nuclear Power Projects and its associated location is not correctly
matched?
(a) Tarapur - Maharashtra
(b) Narora - Uttar Pradesh
(c) Kaiga - Karnataka
(d) Kakarapara - Tamil Nadu
Answer: D
The important nuclear power projects are:
Tarapur (Maharashtra)
Rawatbhata near Kota (Rajasthan)
Kalpakkam (Tamil Nadu)
Narora (Uttar Pradesh)
Kaiga (Karnataka)
Kakarapara (Gujarat)

Q: Which of the following Industrial Regions are classified as Major Industrial Regions?
1. Bengaluru-Tamil Nadu Region
2. Jaipur-Ajmer Region
3. Amritsar-Ludhiana Region
4. Hugli Region
Select the correct answer using the code given below.
(a) 1, 2 and 4 only
(b) 1 and 3 only
(c) 1 and 4 only
(d) 3 and 4 only
Answer: C
Hugli Region and Bengaluru -Tamil Nadu Region are classified as Major Industrial Regions

© APTI PLUS ACADEMY FOR CIVIL SERVICES


Q: Consider the following statements with reference to an industrial region in India:
1. The region owes its development to the discovery of coal in the Damodar Valley.
2. Densely populated surrounding regions provide cheap labour and Hugli region provides vast market
for its industries.
3. Some of the important centres are Rourkela and Durgapur.
Identify the region.
(a) Vishakhapatnam-Guntur Region
(b) Chotanagpur Region
(c) Hugli Industrial Region
(d) Gujarat Industrial Region

Answer: B
Option (b) is the correct answer. Chotanagpur Region -This region owes its development to the
discovery of coal in the Damodar Valley and metallic and non-metallic minerals in Jharkhand and
northern Orissa.
Six large integrated iron and steel plants at Jamshedpur, Burnpur- Kulti, Durgapur, Bokaro and Rourkela
are located within this region.

Q: Which of the following factors influence the location of an industry?


1. Access to agglomeration Economies
2. Access to power/energy
3. Government Policies
4. Access to Transportation facilities
Select the correct answer using the code given below.
(a) 1 and 2 only
(b) 2, 3 and 4 only
(c) 1 and 4 only
(d) 1, 2, 3 and 4

Answer: D
Statement 1 is correct. Many industries benefit from nearness to a leader industry and other
industries.
Savings are derived from the linkages which exist between different industries.
Statement 2 is correct. Industries which use more power are located close to the source of the energy
supply such as the aluminum industry.

© APTI PLUS ACADEMY FOR CIVIL SERVICES


Statement 3 is correct. Governments adopt 'regional policies' to promote 'balanced' economic
development and hence set up industries in particular areas.
Statement 4 is correct. Speedy and efficient transport facilities to carry raw materials to the factory
and to move finished goods to the market are essential for the development of industries. The cost of
transport plays an important role in the location of industrial units.

Q: Consider the following pairs:


Integrated Steel Plant : Associated river
1. Tata Iron and Steel Plant : Subarnarekha
2. Indian Iron and Steel Company : Barakar
3. Rourkela Steel Plant : Bhadravati
Which of the pairs given above is/are correctly matched?
(a) 1 and 2 only
(b) 1 and 3 only
(c) 2 only
(d) 1, 2 and 3
Answer: A
The Tata Iron and Steel plant - Water is obtained from the Subarnarekha and Kharkai rivers.
The Indian Iron and Steel Company (IISCO) - Water is obtained from the Barakar River, a tributary of
the Damodar.
Visvesvaraiya Iron and Steel Works Ltd. (VISL) - Water is obtained from the Bhadravati river.
Rourkela Steel Plant - Water is obtained from the Koel and Sankh rivers.

Q: Woolen industry in India is not as developed as cotton textiles. Which of the following reasons is/are
responsible?
1. Tropical climate
2. Poor quality of indigenous wool
3. Growing competition from synthetic fibres
Select the correct answer using the code given below.
(a) 1 and 2 only
(b) 2 and 3 only
(c) 3 only
(d) 1, 2 and 3
Answer: D
The woolen industry is not developed in India due to the following reasons:
India being a tropical county, woolen clothes are required only in northern India during the winter
months, hence there is not much demand.
Good quality of wool has to be imported due to the poor quality of indigenous wool.
Growing competition from synthetic fibres. They are gradually replacing the woolen industry.
Other reasons include - woolen mills are scattered all over the country. So the mill owners are unable
to work jointly for its improvement.

© APTI PLUS ACADEMY FOR CIVIL SERVICES


Q: Which of the following raw materials used in different industries is a "pure" raw material which does
not lose weight in the manufacturing process?
(a) Iron Ore
(b) Coal
(c) Cotton
(d) Sugarcane
Answer: C
Option (c) is the correct answer. Cotton is a "pure" raw material which does not lose weight in the
manufacturing process.
In iron and steel industries, iron ore and coal both are weight
-losing raw materials. In sugar industry,
Sugarcane is a weight-losing crop.

Q: Which among the following are foot loose industries?


(a) Sugar industry
(b) Electronics industry
(c) Iron and Steel industry
(d) Chemical fertilizers industry
Answer: B
Foot loose industries can be located n ia wide variety of places. They are not dependent on any
specific raw material, weight losing or otherwise. They largely depend on component parts which can be
obtained anywhere. They produce in small quantity and also employ a small labour force. These are
generally not polluting industries.
The important factor in their location is accessibility by road network. These industries often have
spatially fixed costs, which means that the costs of the products do not change despite where the
product is assembled.
Electronics industries are some examples of footloose industries.

Q: Which of the following is not a Maha-Ratna industry?


a. GAIL
b. Coal India Limited
c. SAIL
d. Airports Authority of India
Answer: d
Airports Authority of India falls under the category of mini ratna category 1.

Q: Which of the following were the problems faced by Jute industry?


1. Inadequate supply of raw materials.
2. Competition from substitutes
3. Obsolete machinery
Select the correct answer using the code given below.
(a) 1 and 2 only
(b) 2 and 3 only
(c) 1 and 3 only
(d) 1, 2 and 3
Answer: D
The main problems being faced by the Jute industry are the following:
Inadequate supply of raw materials -After independence, 70 per cent of the jute producing areas went
to Bangladesh. Thus, the jute industry suffers from inadequate supply of raw jute.

© APTI PLUS ACADEMY FOR CIVIL SERVICES


Competition from substitutes -Paper, plastics, cloth and hemp have emerged as substitutes and are
available at cheaper rates in the world market.
Obsolete machinery -Many jute mills are uneconomical because of outdated machinery. Other
reasons are labour problem and irregular supply of raw jute.

Q: Consider the following statements about Cotton textile industry:


1. Cotton textile mill sector is highly capital intensive.
2. Powerloom sector is more labour intensive than Handloom sector.
Which of the statements given above is/are correct?
(a) 1 only
(b) 2 only
(c) Both 1 and 2
(d) Neither 1 nor 2

Answer: A
Cotton textile industry has three sub -sectors i.e. handloom, powerloom and mill sectors.
Handloom sector is labour -intensive and provides employment to semi-skilled workers. It requires
small capital investment. The power loom sector introduces machines and becomes less labour
intensive.
Therefore, statement 2 is not correct.
Cotton textile mill sector is highly capital intensive and produces fine clothes in bulk. Therefore,
statement 1 is correct.

Q: Consider the following pairs:


City Prominent Industry
1. Pittsburg : Iron and Steel
2. Manchester : Cotton textile
3. Detroit : Automobile
Which of the pairs given above is/are correct?
(a) 1 and 2 only
(b) 2 only
(c) 1 and 3 only
(d) 1, 2 and 3

Answer: D
Pittsburgh is a city in Pennsylvania in the United States .It is known for iron and steel industry, this
region has also shifted to high technology, robotics, health care, nuclear engineering, tourism,
biomedical technology, finance, education, and services.
Manchester of United kingdom is known for cotton textile manufacture since the Industrial
Revolution.
Detro it area emerged as a significant metropolitan region within the United States due to its
emergence as a major automobile industry. Automobiles companies like Chevrolet has their
headquarter in Detroit.

© APTI PLUS ACADEMY FOR CIVIL SERVICES


Q: Which of the following industries are usually located near the raw materials?
1. Pulp Industry
2. Copper Smelting
3. Sugar Mills
4. Pig Iron industries
(a) 1, 2, 3
(b) 2, 3, 4
(c) 1, 3, 4
(d) 1, 2, 3, 4
Answer: d
Industries using weight losing materials are located in the regions where raw materials are located.

Q: Which of the following statements are correct?


1. Cheap water transport has facilitated the jute mill industry along the Hugli.
2. Sugar, cotton textiles and vegetable oils are footloose industries.
3. Port towns in India have attracted industries.
(a) 1, 2 Only
(b) 1, 3 Only
(c) 2, 3 Only
(d) All the above

Answer: b
Footloose industries are not dependent on any specific raw material, weight losing or otherwise. The
important factor in their location is accessibility by road network. ex. Computer chip industries,
assembling of components and parts.

Q. Match the following:


1. Heavy Metallurgical Industries
2. Plantation agriculture & Hydropower
3. Jute Industry
4. Cotton Textile
(a) Mumbai-Pune Industrial Region
(b) Hugli Industrial Region
(c) Kollam-Thiruvananthapuram Region
(d) Chotanagpur Region
Choose the correct code:
A. 1-c 2-d 3-a 4-b
B. 1-d 2-c 3-b 4-a
C. 1-b 2-c 3-d 4-a
D. 1-d 2-b 3-c 4-a

Answer: b
Mumbai-Pune Industrial Region: Cotton Textile
Hugli Industrial Region: Jute Industry
Kollam-Thiruvananthapuram Region: Plantation agriculture & Hydropower
Chotanagpur Region: Heavy Metallurgical Industries

© APTI PLUS ACADEMY FOR CIVIL SERVICES


Q: Which of the following ports is popularly known as "Queen of the Arabian Sea" and has an
advantageous location being close to the Suez-Colombo route?
(a) Mumbai Port
(b) Marmagao Port
(c) Tuticorin Port
(d) Kochchi Port

Answer: D
Kochchi Port, situated at the head of Vembanad Kayal, popularly known as the 'Queen of the Arabian
Sea', is also a natural harbour. This port has an advantageous location being close to the Suez-Colombo
route. It caters to the needs of Kerala, southern-Karnataka and south western Tamil Nadu.

Q: Consider the following pairs:


National Waterway Associated River
1. NW 1 : Ganga
2. NW 2 : Brahmaputra
3. NW 3 : Godavari
Which of the pairs given above is/are correctly matched?
(a) 1 only
(b) 1 and 2 only
(c) 2 and 3 only
(d) 1, 2 and 3

Answer: B
Pair 1 is correctly matched. NW 1 (Allahabad -Haldia stretch (1,620 km))- It is one of the most
important waterways in India, which is navigable by mechanical boats up to Patna and by ordinary boats
up to Haridwar. It is divided into three parts for developmental purposes- (i) Haldia-Farakka (560 km), (ii)
Farakka- Patna (460 km), (iii) Patna- Allahabad (600 km).
Pair 2 is correctly matched. NW 2 (Sadiya -Dhubri stretch (891 km))- Brahmaputra is navigable by
steamers up to Dibrugarh (1,384 km) which is shared by India and Bangladesh.
Pair 3 is incorrectly matched. NW 3 (Kottapuram -Kollam stretch (205 km))- It includes 168 km of west
coast canal along with Champakara canal (23 km) and Udyogmandal canal (14 km).

Q: Arrange the following sea ports of India in the order of their location from north to south.
1. Paradip
2. Haldia
3. Marmagao
4. Chennai
Select the correct answer using the code given below.
(a) 2-1-4-3
(b) 2-1-3-4
(c) 2-3-1-4
(d) 3-2-1-4
Answer: B
The correct order is: Haldia (west bengal) - Paradip (odisha) - Marmagao (Goa) - Chennai (Tamil nadu)

© APTI PLUS ACADEMY FOR CIVIL SERVICES


Q: The density of roads (length of roads per 100 square km of area) varies from 10.48 km to 526.9 km in
India. Which of the following is/are the reasons behind?
1. Nature of terrain
2. Level of economic development
3. Predominance of agricultural activities
Select the correct answer using the code given below.
(a) 1 only
(b) 1 and 2 only
(c) 2 and 3 only
(d) 1, 2 and 3

Answer: B
The distribution of roads is not uniform in the country. Nature of terrain and the level of economic
development are the main determinants of density of roads. Therefore, both 1 and 2 are correct.
Predominance of agricultural activities is not the reason behind varying density or low density of
roads.
In fact it should have led to more density of roads.

Q: Which of the following roads constitute the maximum length of roads in India?
(a) National Highways
(b) State Highways
(c) District Roads
(d) Rural Roads

Answer; D
Rural or Village roads account for over 58 per cent of the total length of roads in the country.
National Highways - 2 per cent
State Highways - 3.5 per cent

Q: Which of the following statement is not correct regarding ports?


(a) The major ports handle about 75 per cent of the country’s oceanic traffic.
(b) Visakhapatnam Port in Andhra Pradesh is a landlocked harbour connected to the sea by a channel
cut through solid rock and sand.
(c) Kochchi Port,situated at the head of Vembanad Kayal, is a natural harbour.
(d) Paradwip Port is situated in the Godavri delta, deepest harbour specially suited to handle very large
vessels.
Answer: d
Paradwip Port is situated in the Mahanadi delta, about 100 km from Cuttack and not Godavari delta.

Q: Pipelines are one of the modes of transportation of various goods. Which of the following goods
is/are transported through pipelines?
1. Natural Gas
2. Coal
3. Iron Ore
Select the correct answer using the code given below.
(a) 1 only
(b) 1 and 2 only

© APTI PLUS ACADEMY FOR CIVIL SERVICES


(c) 2 and 3 only
(d) 1, 2 and 3

Answer: D
Pipelines are one of the modes of transportation along with land, water and air. Other than
passengers they carry majority of items in liquified forms like:
Crude oil, natural gas and petroleum
Coal (in liquified form)
Ores (like iron, copper, Bauxite etc) through slurry pipeline

Q: Which of the following can be developed for navigation through inland waterways?
1. Rivers
2. Canals
3. Lakes
4. Coastal areas
Select the correct answer using the code given below.
(a) 1 and 2 only
(b) 2 and 4 only
(c) 1, 2 and 3 only
(d) 1, 2, 3 and 4

Answer: D
o Rivers, canals, lakes and coastal areas are all used as waterways , provided they have adequate depth
,width and river flow for navigability.
o Rivers are the only means of transport in dense forest like Congo etc.
o Lakes like Huron and Erie supports forward and backward linkages of prairies through waterways over
it. This has substantially reduced the cost of transportation of cereals in the region.
o Coastal areas also helps in transportation of goods to adjacent regions.

Q: Consider the following pairs : [2014 ]


National Highway Cities connected
1. NH 4 Chennai and Hyderabad
2. NH 6 Mumbai and Kolkata
3. NH 15 Ahmedabad and Jodhpur
Which of the above pairs is/are correctly matched?
(a) 1 and 2 only
(b) 3 only
(c) 1, 2 and 3
(d) None
Answer: d
NH 4 is a major National Highway in Western and Southern India. It connects Mumbai with Chennai.
NH6, runs through Gujarat, Maharashtra, Chhattisgarh, Odisha, Jharkhand and West Bengal state in
India. It connects Surat with Kolkata.
NH 15 is a major National Highway in Western and Northwestern India. NH 15 connects Samakhiali in
Gujarat with Pathankot in Punjab.

© APTI PLUS ACADEMY FOR CIVIL SERVICES


Q. Consider the following statements regarding Indian roads:
1. The length of the national highways is greater than the state highways.
2. The length of the panchayati raj roads is greater than the municipal roads.
Which of the above statements are correct?
(a) Both 1 and 2
(b) None
(c) 1 only
(d) 2 only

Answer: d
The length of highways or roads follow the order: Panchayati roads>urban roads> state highway>
national highway

Q. Consider following statements about National Highways:


1. The National Highways constitute only two per cent of the total road length but carry 40 per cent of
the road traffic.
2. NH 7 is the longest highway in the country.
Which of the statements given above is/are correct
(a) Only 1
(b) only 2
(c) Both 1 and 2
(d) Neither 1 nor 2.

Answer: c
Statement 1 correct:
Statement 2 Correct: National Highway 7 is 2,369 km long road is the longest National Highway of India
and one of the major road network between south and north India. The National Highway 7 cross 6
Indian states and connects major
Indian cities like Varanasi,Nagpur, Hyderabad,Bangalore and Madurai. Some of the part of NH7 is
selected as North-South Corridor and Golden Quadrilateral highway projects.
Q:Inland Ports are located away from the sea coast and are linked to the sea through a river or a canal.
Which of the following port is an example of "Inland port"?
(a) Abadan Port
(b) Singapore Port
(c) Kolkata Port
(d) Rotterdam Port
Answer: C
 Inland Ports are located away from the sea coast. They are linked to the sea through a river or a canal.
Memphis located on the river Mississippi, Mannheim and Duisburg on Rhine in Germany and Kolkata
located on the river Hoogli are some of the inland ports. Therefore, (c) is the correct answer.

Q: Activities that focus on the creation, rearrangement and interpretation of new and existing ideas,
data interpretation and the use and evaluation of new technologies are known as:
(a) Secondary activities
(b) Quinary activities
(c) Tertiary activities
(d) Primary activities

© APTI PLUS ACADEMY FOR CIVIL SERVICES


Answer: B
While Quaternary activities involve the collection, production and dissemination of information.
Quinary activities focus on creation, re-arrangement and interpretation of new and existing ideas, data
interpretation and the use and evaluation of new technologies and are performed by the highest level of
decision makers or policy makers.

Q: Which of the following are classified as Tertiary activities under service sector?
1. Trade
2. Decision Making
3. Policy formulation
4. Communication
Select the correct answer using the code given below.
(a) 1, 3 and 4 only
(b) 1 and 4 only
(c) 2 and 3 only
(d) 1, 2, 3 and 4

Answer: B
Activities under Services Sector-
Tertiary activities- Trader and commerce, Transport, Communication.
Quaternary activities - Information based, R&D based.
Quinary activities - specialists, decision makers, consultant, policy formulation.
Therefore, (b) is the correct answer.

Q: Consider the following statements regarding Tertiary activities:


1. It involves the commercial output of services rather than the production of tangible goods.
2. It relies more heavily on specialized skills and experience rather than on the production techniques.
Which of the statements given above is/are correct?
(a) 1 only
(b) 2 only
(c) Both 1 and 2
(d) Neither 1 nor 2
Answer: C
Statement 1 is correct. Tertiary activities involves both production and exchange. The production
involves 'provision' of services that are 'consumed'. Tertiary activities, therefore, involve the commercial
output of services rather than production of tangible goods.
Statement 2 is correct. The main difference between secondary activities and tertiary activities is that
the expertise provided by services relies more heavily on specialised skills, experience and knowledge of
the workers rather than on the production techniques, machinery and factory processes.

Q. The sea coast of which one of the following states has become famous as a nesting place for the giant
Olive Ridley turtles from South America? [2002]
(a) Goa
(b) Gujarat
(c) Orissa
(d) Tamil Nadu
Answer(c)

© APTI PLUS ACADEMY FOR CIVIL SERVICES


The sea coast of Orissa is famous as a nesting place for giant olive Ridley turtles from South America.
Q. What is the correct sequence of the rivers–Godavari, Mahanadi, Narmada and Tapi in the descending
order of their lengths? [2003]
(a) Godavari–Mahanadi–Narmada–Tapi
(b) Godavari–Narmada–Mahanadi–Tapi
(c) Narmada–Godavari–Tapi-Mahanadi
(d) Narmada–Tapi–Godavari–Mahanadi
Answer (b)
The correct sequence of the river in descending order of their lengths are Godavari (1465 km), Narmada
(1312 km), Mahanadi (858 km) and Tapti (724 km).

Q. Due to their extensive rice cultivation, some regions may be contributing to global warming. To what
possible reason/reasons is this is attributable?[2010]
1. The anaerobic conditions associated with rice cultivation cause the emission of methane.
2. When nitrogen based fertilizers are used, nitrous oxide is emitted from the cultivated soil.
Which of the statements given above is/are correct?
(a) 1 only (b) 2 only
(c) Both 1 and 2 (d) Neither 1 nor 2
Answer: (c)
Both the statements are correct

Q. Consider the following pairs: [2008]


Tributary River: Main River
1. Chambal Narmada
2. Sone Yamuna
3. Manas Brahmaputra
Which of the pairs given above is/are correctly matched?
(a) 1, 2 and 3
(b) l and 2 only
(c) 2 and 3 only
(d) 3 only
Answer. (d) Tributaries of Brahmaputra in India the Manas, Pagladiya, Puthimari, Dhanisri, Jia Bhariti
and Subansiri. Manas is a tributary of Brahmaputra
Chambal is the chief tributary of Yammuna and sone is a tributary of Ganga.

Q. The approximate representation of land use classification in India is : [2010]


(a) Net area sown 25%, forests 33%, other areas 42%
(b) Net area sown 58%, forests 17%, other areas 25%
(c) Net area sown 43%; forests 29%, other areas 28%
(d) Net area sown 47%, forests 23%, other areas 30%
Answer (d)
The approximate representation of land use is classified as net area sown 46%, forests 23%, other areas
30%.

Q.Which of the following is/are the characteristic/ characteristics of Indian coal? [2013 - I]
1. High ash content
2. Low sulphur content
3. Low ash fusion temperature

© APTI PLUS ACADEMY FOR CIVIL SERVICES


Select the correct answer using the codes given below.
(a) 1 and 2 only
(b) 2 only
(c) 1 and 3 only
(d) 1, 2 and 3
Answer (a)
Indian coal has high ash content and low calorific value. It has low sulphur and low phosphorous content
but high ash fusion temperature.

Q. Consider the following crops: [2013 - I]


1. Cotton
2. Groundnut
3. Rice
4. Wheat
Which of these are Kharif crops?
(a) 1 and 4
(b) 2 and 3 only
(c) 1, 2 and 3
(d) 2, 3 and 4
Answer (c)
Rice is the main kharif crop and groundnut and cotton are also the kharif crops wheat is a rabi.

Q. The terms 'Agreement on Agriculture', 'Agreement on the Application of Sanitary and Phytosanitary
Measures' and 'Peace Clause' appear in the news frequently in the context of the affairs of the [2015-I]
(a) Food and Agriculture Organization
(b) United Nations Framework Conference on Climate Change
(c) World Trade Organization
(d) United Nations Environment Program
Answer: (c)
'Agreement on Agriculture', 'Agreement on the Application of Sanitary and Phytosanitary Measures' and
'Peace Clause' are related to World Trade Organisation

Q: With reference to the Quaternary activities, which of the following statements is/are not correct?
1. Unlike the tertiary activities, quaternary activities cannot be outsourced.
2. Quaternary activities are tied to resources and are not affected by the environment or localised
market.
Select the correct answer using the code given below.
(a) 1 only
(b) 2 only
(c) Both 1 and 2
(d) Neither 1 nor 2
Answer: C
Both the statements are incorrect with reference to quaternary activities.
Like some of the tertiary fun ctions, quaternary activities can also be outsourced. Because they are not
tied to resources, affected by the environment, or necessarily localised by market.

© APTI PLUS ACADEMY FOR CIVIL SERVICES


Q: Consider the following pairs:
Type of worker Associated Activity
1. Blue Collar : Primary Activities
2. Gold Collar : Quinary Activities
3. Red Collar : Workers in industries
Which of the pairs given above is/are correctly matched?
(a) 1 and 3 only
(b) 2 only
(c) 2 and 3 only
(d) 1, 2 and 3
Answer: B
Red Collar - Primary Activites
Gold Coll ar - Quinary
White Collar - Professionals in High Technology industry
Blue Collar - Workers in industries

Q: Which of the following activities is/are primary activities?


1. Mining
2. Quarrying
3. Construction
Select the correct answer using the code given below.
(a) 1 and 2 only
(b) 1 and 3 only
(c) 2 only
(d) 1, 2 and 3
Answer: A
1 and 2 are correct. Human activities which generate income are known as economic activities.
Economic activities are broadly grouped into primary, secondary, tertiary and quaternary activities.
Primary activities are directly dependent on environment as these refer to utilisation of earth‘s
resources such as land, water, vegetation, building materials and minerals. It, thus includes, hunting and
gathering, pastoral activities, fishing, forestry, agriculture, and mining and quarrying.
3 is not correct. Secondary activities add value to natural resources by transforming raw materials into
valuable products. Secondary activities, therefore, are concerned with manufacturing, processing and
construction (infrastructure) industries

© APTI PLUS ACADEMY FOR CIVIL SERVICES

You might also like